Conference Notes       7-16-2013     Todays’ focus: ED  CODEs

There are images in this document.  If you don’t see them, scroll to the bottom and click “read in browser”

McKean     Intial Mangement of Sick Patients

Case 1 Hyperkalemia  most rapidly identified on EKG.   Treat with calcium, insulin/glucose, bicarb, kayexelate

Hyperkalemic EKG 

Case 2   Decompensated CHF   Girzadas comment:  I follow Harwood’s Maxim that if you are intubating an acute CHF exacerbation patient you have failed.   These are patients that if you aggressively treat with sublingual and hi dose IV ntg, bipap and alittle bit of diuresis you can turn them around in 30-60 minutes.   Harwood agreed.

Case 3 Septic/hypotensive patients need aggressive IV fluids  30ml/KG in short order.  IV antibiotics are also needed.  Elise comment: In septic patients do a complete exam for possible sources.  Look specifically for  pneumonia/uti/abdominal infectious process/cns infectious signs/skin pressure ulcerations or cellulitis or necrotizing fasciitis.   Tekwani comment: make sure you get 2 blood cultures.

C Kulstad comment: For all these really sick patients make sure your attending is in the room with you.  These patients require aggressive care including the ED attending at bedside.  

Kettaneh    Intraosseous   Lines

IO lines are fast, they work 97% of the time, cost is about $100 and you can give fluids and medications and get blood samples.   It can be a bridge access device prior to getting a central line.  Consider in status epilepticus and in sepsis patients with access problems.  IO lines give access to central circulation.

Contraindications: Proximal fx,  overlying cellulitis, previous IO line in the same bone

Pain with infusion is a common side effect.  2.0 ml of 2% of cardiac lidocaine infused into the IO line can make  Infusions less painful.

Proximal humerus access site:  Technique to find best site is put pt’s hand on umbilicus.  Palpate for notch on the humeral head.  Lovell and Girzadas comment: It’s ok to just palpate for boney prominence of humerus that has the least soft tissue overlying it.  This is not rocket science, the goal is to get access in the bone.  It doesn’t have to be in a perfect position.

 

 

Proximal Humerus SITE/Angle  for IO Line

 

Proximal tibia access site: 2 finger breadths below tibial tuberosity on medial aspect.

There is some data to suggest the proximal humerus site has more rapid access to the central circulation.  However, the proximal tibial site has rapid transport to central circulation also.   Probably should choose access site based on ease of access.

Lines on the  IO needle are at 5mm demarcations.  You should get needle to surface of bone prior to drilling into the bone.  When needle is on bone surface, you should have at least one line on the needle visible outside the skin.  Seeing that line assures you that you have enough needle length to successfully access the bone and make sure it is secure.

Aspiration of blood confirms IO placement.   Kasia comment : No blood back on first aspiration does not necessarily mean you are not in the bone.   Harwood comment: You can use this aspirated blood for labs such as type and screen/HGB/Bun and Cr.   Elise comment: You don’t always need to numb up the skin in urgent situations. 2.0 ml of 2% of cardiac lidocaine infused into the IO line can make  Infusions less painful.  Elise comment: 2-3 ml from an amp of lidocaine should do the trick.  Painful infusion can be mistaken for improper IO placement but it really means that you are infusing into the bone and it is painful.  You need to infuse lido as described to decrease pain.  Bone pain with infusions is actually a sign you are in the bone properly in an awake patient.  Flush lines prior to infusions.  You may need pressure bag to get fluids/drugs infused.   

Potassium WBC, Platelet and bicarb are not reliable to test with IO obtained blood. 

You can give virtually any med or fluid or transfusion through IO.  You can even give pressors and TPA.

IO lines are recommended to be removed by 24 hours.   Remove by hooking up a 10ml syringe.  Turn the needle clockwise and pull back gently.

Elise comment: In a code situation with 1or 2 failed peripheral IV attempts go right to IO line before central line.

Purnell            ABG’s

3 step approach

  1. Calculate the anion gap (identify an anion gap acidosis)  AG=Na –(Hco3 +CL).   NL AG=15.   Lithium or low albumin can shrink the gap.  In that light, an end-stage cirrhotic with normal or high gap is in metabolic trouble.
  2. Calculate the delta gap (identify hidden metabolic abnormalities).   Change in the AG should equal the change in HCO3.  For every 1 change in AG, the HCO3 should go down by 1.  Normal AG=15,  Normal HCO3=24.  They should both normall change in a 1:1 ratio.  If not, some metabolic derangement is going on.    If bicarb is too high you have a primary metabolic acidosis.   If bicarb is too low then you have a non-anion gap metabolic acidosis.
  3. Apply the rule of 15 (identify respiratory acidosis/alkalosis)  HCO3 + 15 should= pCO2 +/-2 and also equal the last two digits of the ph.  If not, another primary respiratory problem is going on.   Pco2 too high=respiratory acidosis,   Pco2 too low=respiratory alkalosis.

Examples:    Ph of 7.25   HCO3=15    PCO2=25    NA=140    Cl=105    

Step 1:  140- (105+15)=20   Thus there is an anion gap acidosis

Step 2:  Change in AG from normal is 5.   Change in bicarb from normal is 9.  Bicarb went down more than expected, thuse we have a non-ion gap acidosis.

Step 3: 15+ 15=30   The PCO2 is too low, thus we have a respiratory acidosis.

We just diagnosed a triple acid/base disorder.

DDX of different acid base abnormalities

High AG Acidosis: CAT MUDPILES is the mnemonic.    CO/CN, AKA, Toluene, Methanol, Uremia, DKA, Paraldehyde, INH?IRON, Lactate, Ethylene glycol, ASA

Non AG Acidosis: HARDUP  Hyperventilation, addisons/acetazolamide, RTA, diarrhea (by far the most common reason), ureteral diversion, P(missed the P in the mnemonic)

Metabolic Alkalosis: dehydrated, cushing syndrome

Respiratory alkalosis: hyperventilation, salicylate poisoning, pregnancy, brain tumors, anemia, high altitude

Respiratory acidosis: Obesity, copd, insufficient ventilation

Herrmann      Non-Invasive Ventilation

Bipap decreases intubations and hospital length of stay.   Non-invasive ventilation means the interface between O2 and patient is a mask, not an ET tube.

  EPAP=PEEP=CPAP

IPAP=pressure support

BIPAP= EPAP +IPAP= CPAP +PS

Funny analogy: CPAP is positive end expiratory pressure which functions basically as a hair dryer in your mouth.

Idications: COPD, CHF, Asthma, pneumonia, atelectasis, neuromuscular disease, palliative care.  Can be considered in any dyspneic patient.

Absolute Contraindications:  need for intubation, decreased level of consciousness,  vomiting or copious oral secretions, facial deformity.

Ventilation improved by IPAP.  Oxygenation improved by EPAP.

BIPAP improves CHF by decreaseing preload and afterload.  Forces fluid out of alveoli. CPAP reduces in-hospital mortality. BIPAP and CPAP reduce intubation with no increased risk of MI.

In COPD, BIPAP improves laminar flow, stents open collapsing airways.  Reduces mortality and intubation.

AVAPS: Averaged volume  assured pressure support.  Gives a specified tidal volume, pressure regulated non-invasive ventilation.  Consider in hypercapnic respiratory failure.

Settings: Standard  12 IPAP/ 5 EPAP,   COPD  15 IPAP/5 EPAP or AVAPS, 

Elise comment: RT’s are not totally comfortable with AVAPS.  Good time to use AVAPS is with the hypercapneic COPD’ers.   It gives them better tidal volumes.

PUtman  STEMI

Posterior MI: Posterior leads only require 0.5mm ST elevation to meet STEMI criteria.

Sgarbossa criteria:  STEMI in the setting of LBBB.  STE that is concordant,   STD that is concordant, STE>5mm that is discordant (less specific)

STEMI management:   If you are in an ED without a cath lab, if you can transfer a patient to a hospital with a cath lab in under two hours your should transfer rather than give thrombolytics.   Other standard treatment for STEMI:   ASA 325 mg.  Plavix 600mg.  Give unfractionated heparin.

Thrombolytics: Harmful for ST depression alone unless a true posterior MI.  If giving lytics, give ASA 325mg,  Plavix 300mg or 75mg if patient is older than 75yo.   Also give unfractionated heparin. 

Beta blockers:  Give within the first 24hours unless CHF, shock, heart block, asthma.  Probably does not need to be given in the ED unless you need to slow heart rate. 

STEMI and Cardiogenic Shock  all need Cardiac Cath and balloon pump irrespective of time delay.

Mckean comment: The big STEMI mimic to have your guard up for is aortic dissection.  Girzadas comment: The one thing that will kill a STEMI patient in the ED is V-Fib.  Be prepared to defibrillate.  Elise/Christine comment: Cardiac surgeons are for the most part OK with plavix being given pretty much as a default to our STEMI patients.  Mistry comment: Don’t worry too much about the duration of a patient’s symptoms prior to arrival in the ED.  You can’t be sure when the angina proceeded to infarct.   Documentation of reason for delay to PCI is critical.

Elise’s added info:

1.  Alteplase (TPA) dosing for acute MI:

 

Accelerated infusion of alteplase for AMI consists of a 15-mg IV bolus followed by 0.75 mg/kg (up to 50 mg) IV over 30 minutes and then 0.5 mg/kg (up to 35 mg) IV over 60 minutes. The maximum total dose is 100 mg for patients weighing more than 67 kg. 

 

 

2.  Plavix dosing if patient already on Plavix and presents with STEMI:

 

I just looked through 2013 AHA STEMI guidelines again, and nothing I can find about folks who are already taking Plavix...like Andrej said, reasonable to re-load them with 600 mg, or can discuss with Cardiology.

 

 

 

3.  Sgarbossa accuracy to predict MI in LBBB.  Criteria are not equally accurate; there are different points for each criteria, the first one most specific/useful.  

 

The three Sgarbossa criteria used to diagnose infarction in patients with LBBB are:

  • ·         Concordant ST elevation > 1mm in leads with a positive QRS complex (score 5)
  • ·         Concordant ST depression > 1 mm in V1-V3 (score 3)
  • ·         Excessively discordant ST elevation > 5 mm

A total score of  ≥ 3 has a specificity of 90% for diagnosing myocardial infarction.

 

Maletich                 Post-Arrest Care

Hemodynamic instability is common after resuscitation.  Have a pressor like norepi ready to go if needed. 

Get them to cath lab. For sure all STEMIs but even 50% of non-STEMI’s wil benefit from cardiac cath after cardiac arrest.   Therapuetic hypothermia has been shown by RCT’s to improve neurologic outcome after resuscitation after shockable rhythms.   Anyone who is not following commands or GCS <8 should get cooled after resuscitation from cardiac arrest.  Cool patient to 33 degrees for 24 hours then rewarm to 37 degrees and control temp for 48 hours. 

Give midazolam to control shivering We also use buspirone, magnesium, and Demerol.  Bradycardia is expected with cooling.  There is cold dieresis. Hypokalemia can develop with hypothermia and it will improve with re-warming.   No need to replete potassium if  3.4 or above. 

 McKean/Elise comment:  Can’t prognosticate patients’ neuro status before 72 hours after therapeutic hypothermia.   Elise comment: There is time urgency for cooling.  Time is brain cells.  Get cooling started ASAP after resuscitation.  Chintan comment: We can’t improve patients’ pre-arrest neuro status with cooling.

Wise    U/S in the Critically Ill Patient

Parasternal long and subxyphoid views are the money shots for evaluating the critically ill patient.

Pericardial effusion: 100ml or less only posteriorly in systole.   >100ml you will see posteriorly and anteriorly on parasternal long view.

 

RV normally should be about 0.5 the size of the LV.  If the RV is similar in size or larger than the LV you have to start considering PE.

Lung U/S is useful to identify pneumothorax, pulmonary edema, and effusions.

Pneumothorax won’t have sliding pleura on B mode and no “beach” in M mode.  

Pulmonary edema is identified with “headlights in the fog” .   Copd is identified  by increased A lines and no headlights in the fog.    Pneumonia will have unilateral or isolated area “headlights in the fog”.

Small Group Workshop    EGDT for the SEPSIS Patient

Critical actions: 

Early identification of sepsis based on 2 or more SIRS criteria and evidence of infection. 

30ml/kg IV fluid bolus within the first hour based on clinical suspicion of sepsis.

Get two blood cultures and lactate in addition to other basic labs

IV antibiotics (Vanco/Zosyn) within an hour

If pt is hypotensive despite fluid bolus or lactate >4  place CVP line.  Give fluid boluses up to a CVP of 8-12

If still hypotensive start IV norepinepherine.

Check SCVO2 and trend lactates.  If SCVO2 low or lactate not improving consider transfusion to get hgb >7, and consider intubation to improve oxygenation and decrease work of breathing.

If still not doing well, can add dobutamine as an inotrope. 

There was a question as to why some clearly septic patients have low lactates.  Elise discussion: In his podcast, Scott Weingart discusses caring for patients with minimally bumped lactates who  are clinically septic.  It's hypothesized that lactate production is driven by surges in catecholamine production, and in some patients, especially the elderly, their ability to produce catecholamines is burnt out, and lactate may not be significantly elevated.  Still need to treat them as septic.

 

Conference Notes 7-9-2013

Conference Notes 7-9-2013

There are 5 images in this document.  If you don't see them, scroll to the bottom and click on "Read in Browser" 

This Tuesday was an Ultrasound Workshop with 2 hours of didactics and 2.5 hours of hands-on workshop.

Lambert                                     U/S  Knobology

Center  your image on the screen and adjust the depth of the image to maximize its size on the screen.  Depth of image is key to making the important objects on the screen best visible.

Every Ultrasound probe can be toggled through 3 modes:

High frequency =high resolution mode

Low frequency=deeper penetration mode

Intermediate frequency= general mode

The toggle switch is at the inferior portion/right side of screen

Visualize your target organ or area in 2 planes to define the boundaries of the object.   If you don’t define the outer boundaries, you can miss things like gallstones or IUP’s.

 

Lambert       7Up Scan   Sonographic Approach to Investigating Shock/PEA

Why?  EP’s correctly identify the cause of shock only 24% of the time.   Healthcare providers often do not correctly identify a carotid pulse.  U/S is accurate for determining intravascular volume.

How do you do it?   RUQ> Subcostal>IVC>Aorta>pelvis>parasternal long axis of heart>Lung

Morrison’s pouch is second lowest area of supine abdomen.  Suprapubic area of pelvis is lowest but it has a small volume and fluid there may escape detection.   So Morrison’s pouch is called by sonographers  the “Money Pouch”.

UCG positive female with fluid in Morrison’s pouch has nearly 100% specificity for ruptured ectopic pregnancy.

Subcostal view should include the liver and the heart.   

Hypovolemia will cause the heart to have a small/collapsing RV and a hyperdynamic LV.

Poor overall contractility suggests cardiogenic shock.   Large RV suggests PE.   Specifically the RV will bow out toward the LV and be of similar size to LV or larger than the LV in a patient with shock from PE.

 

Hyperdynamic LV function in setting of shock is likely to indicate sepsis as the cause of shock.

Proximal  IVC  views are helpful for gauging volume status.  Hypovolemia causes decrease in IVC diameter >50% with inspiration.

Landmarks for aorta views are spine/IVC/aorta.   Aorta diameter >3cm =aneurysm.   The larger the aneurysm the more likely it is to be ruptured.   In a patient in shock without abdominal or back pain and a 3cm aorta.  The AAA may not be the cause of shock.  Smaller aneurysms are less likely to rupture.

On Parasternal long view, good contractility is identified by seeing the anterior mitral valve leaflet strike the ventricular septum.

Lung views can help identify pneumothorax or chf.  Pneumothorax shows no sliding of pleura and no moving comet tails.  With M mode, normal lung has sky/beach appearance.   With pneumo, M mode shows no beach.   

 

 

 

 Large comet tails “headlights in the fog” is a good sign of CHF.   Girzadas comment: I find the “headlights in the fog “ sign very specific for CHF.   Mike agreed with this.

 

 

 

Please consider donating to our EM Foundation to benefit resident education/development.  Thanks for your consideration/generosity.
/em-foundation/
 

 

 

Conference Notes 7-2-2013

Conference Notes 7-2-2013   There are images in this document.  If you don't see them scroll to the bottom and click on "read in browser"

Herrmann/Harwood    Oral Boards

Case 1: 60yo man with botulism.  Critical actions:  Evaluate respiratory status, give trivalent antitoxin.  Military has heptavalent antitoxin that is not yet commercially available.   Babies should not receive antitoxin, instead they get human botulism immune globulin (BabyBIG).     Most cases of botulism are infantile (72%).  Another 25% of botulism cases (mostly adults)  are due to improperly canned food.  The rest of the cases are due to wound infections. There are about 150 cases of botulism in the US per year.  Botulinum toxin blocks the release of acetyl choline at the neuromuscular junction resulting in weakness.   Physical exam shows ptosis, dysconjugate gaze, dilated pupils, weakness.

    

Google image

 Cranial nerve involvement most commonly marks the onset of symptomatic illness and can include blurred vision (secondary to fixed pupillary dilation and palsies of cranial nerves III, IV, and VI), diplopia, nystagmus, ptosis, dysphagia, dysarthria, and facial weakness. Descending muscle weakness usually progresses to the trunk and upper extremities, followed by the lower extremities. Urinary retention and constipation are common resulting from smooth muscle paralysis. Occasionally paresthesias and asymmetric limb weakness are seen [37]. Similar to infant botulism, respiratory difficulties (eg, dyspnea) requiring intubation and mechanical ventilation are common, caused by diaphragmatic paralysis, upper airway compromise, or both. Despite the evidence of neurologic involvement, cerebrospinal fluid analysis is normal.  (Up to date_)

     

Picture of bilateral ptosis due to botulism

Elise comment: If you give antitoxin to infants they potentially may develop severe anaphylaxis to antitoxin and they can develop life-long sensitization to equine proteins.  

Case2:  11yo male stabbed in chest by his mom.  Pt had a sucking chest wound.  Pt had a pericardial tamponade.  Pt lost vitals and required ED thoracotomy.  Pericardial sac was opened, clots were removed, and heart was delivered.  RV wound was stapled closed.   Critical actions: Place a chest tube initially, intubate, do a thoracotomy, get them to the OR.    Harwood made the point that if you have a tension pneumo with a open chest wound, you can often relieve the tension by removing any dressing on the wound so air can be released.  Harwood comment: Stab to the chest with no pneumo and no suspected cardiac or vascular injury  can be managed with a repeat 6 hour CXR. If still no pneumo, patient can go home.   For an ER doc the best chance you have to save a patient with a thoracotomy is a stab wound to the chest causing pericardial tamponade.  If you can relieve the tamponade and staple the heart, patients can survive this injury.   Girzadas comment:  AT one point in this case, the echo showed pericardial fluid, BP was 80/40, and pt was mentating.  Should you perfrom pericardiocentesis, crack the chest in the ER, or go to OR.   Harwood: If you are at a Trauma Center, Just get the patient to the OR.  IF you are at an ED where you will have to transfer patient, pericardiocentesis is a reasonable temporizing maneuver.   Only perform an ED thoracotomy if patient loses vitals or goes into extremis.

Case 3:  Pt presented with severe muscle and abdominal cramps while working in hot weather.   Pt had heat cramps.  This is due to a combination of salt loss, fluid loss and muscle fatigue.   Harwood comment: Old southern remedy is pickle juice. It has fluid/salt/some carbs.  Critical actions:  IV normal saline is the treatment.

Nierzwicki    Dental emergencies

Dentoalveolar trauma:  With any dental trauma, make sure there is not a dental FB or boney FB somewhere in the mouth or pharynx.   Normal adults have 32 teeth.  With wisdom teeth removed, the normal adult has 28 teeth.   Fun fact: If people don’t get their wisdom teeth removed, they are at higher risk for low birth weight babies (not sure why) and unstable coronary plaques (due to strep colonization of wisdom teeth and subsequently the coronary artery plaque).   

Ellis fracture’s: Class 1 is through enamel.  Class 2 is through enamel and dentin,  Class 3 is through enamel,dentin,pulp.   Dr. Nierzwicki said the main ED management is to make sure the patient did not aspirate the fracture fragment.   You don’t need to patch over the fractured tooth in the ED.   Even an ellis class3 fracture is not an emergency.  Consider antibiotics and pain meds.  That’s it.   Girzadas comment: This is great news!  EM texts  say Ellis class 3’s require emergent calcium hydroxide coverage. This is basically not super easy to do.  So hearing that it is not emergently necessary is great.  Elise comment: Do you want us to mix up calcium hydroxide and cover an Ellis 3 dental fracture?   Nierzwicki says no.  The main issue is they need follow up with a dentist or oral surgeon.  This of course is difficult for people with no dental coverage.

  Loose teeth:  If tooth is moveable in ant/post plane or ant/post and lateral plane it probably will heal and remain stable.  If it is moveable in 3 planes (a/p, lateral, up and down in the socket) the tooth will need stabilization.   The ER doc should not try to improve the luxation of an injured tooth.  Leave it for the dentist or oral surgeon.  Even the oral surgeons don’t attempt to improve intrusion injuries (impacted teeth).  They let them come down naturally.  Elise comment: For subluxations injuries do you want the ER doc to do anything besides referral?   Nierzwicki said no, unless the tooth is dangling and is at risk of being aspirated or lost.   As for imaging, if a single tooth is injured and the tooth is relatively stable and the bone around the injured tooth is stable, then you don’t need to get xrays in the ED.   For a completely avulsed tooth, you want to get it back in the socket within 2 hours.   To transport tooth Hanks solution is the best transport medium, followed by saliva, milk and saline.  The ER doc can rinse the tooth with saline. Don’t scrub or rub the tooth.   The tooth needs to be splinted in place for 10 days.  Girzadas comment:  Everyone has sandwich bags at home.  Can the parents put the tooth in a baggie and spit a few times in the bag?  Nierzwicki said sure, or put milk in the bag.   To differentiate between an alveolar fracture and a Le Fort 1 fracture, put your fingers posterior to the upper molars and apply anterior force.  If all the upper teeth move forward, you have a Le fort 1 Fracture.  

Carlson          Lithium and Valproate Toxicity

Lithium   Discovered in 1818 and initially used for gout and kidney stones.   It was used as a table salt and was an ingredient in 7UP!    Commonly used for bipolar disorder and has possible utility for cluster and migraine headaches, impulsive behavior, neutropenia and etohism. 

A way to think about lithium is that it acts like potassium in the body. It is a positive cation.  It’s therapeutic mechanism is unknown.   Adverse effects include decreased ability to concentrate urine to the point of nephrogenic DI.  It also causes hypothyroidsm and hyperparathyroidism.   Long term lithium treatment commonly causes goiter.   Lithium can cause nonspecific  t wave changes and U waves. Girzadas comment: U waves are not intuitive if you think of lithium as analogous to K+.  Andrea said U waves were noted in case reports.  There is some concern about teratogenicity (specifically ebstein’s anamoly of the tricuspid valve and RV).

Don’t  use NSAID’s in patients with lithium.  Lithium is excreted by a prostaglandin dependent renal mechanism. NSAID’s inhibit this mechanism.  This is a known interaction and the emergency physician should be aware of it.   Postassium sparing diuretics can increase lithium levels.   Lithium + SSRI can possibly cause serotonin syndrome.  Lithium +antipsychotic medications  can possibly cause NMS.

Toxicology Factoid:  Volume of distribution <1 means the toxin is mostly limited to the blood compartment and can be dialyzed well.  If volume of distribution is >1 that means the toxin is distributed throughout the body and is not well dialyzed.    The other thing that makes a toxin dialyzable is low protein binding in the bloodstream.  

Lithium has a volume of distribution <1 and is not significantly protein bound in blood stream.  Hence dialysis is an effective treatment for significant lithium toxicity.

Toxicity of Lithium can look like thyrotoxicosis or gastroenteritis.  Can look like serotonin syndrome. 

  • §  Gastrointestinal   Patients with acute lithium toxicity often develop and present with symptoms of nausea, vomiting, and diarrhea. If vomiting and diarrhea are severe, dehydration and compromised renal function can develop, impairing the ability to excrete lithium and exacerbating lithium toxicity.
  • §  Cardiac  Although lithium toxicity can cause changes in the electrocardiogram (ECG), dangerous arrhythmias or other important clinical effects are rare [14,15]. Prolonged QTc intervals and bradycardia have been reported [15-17]. Lithium poisoning is not associated with elevations in cardiac biomarkers or left ventricular dysfunction
  • §  Neurologic  Neurologic findings develop late in acute lithium poisoning because time is required for the drug to be absorbed and to penetrate the central nervous system (CNS). Potential neurologic symptoms and signs include sluggishness, ataxia, confusion or agitation, and neuromuscular excitability, which can manifest as irregular coarse tremors, fasciculations, or myoclonic jerks. Severe lithium intoxication can lead to seizures, nonconvulsive status epilepticus, and encephalopathy.  (Up to Date)

Get serial lithium levels until you are sure the level is going down.  Levels >4 are usually dialyzed.   All patients should receive IV hydration.  Kayexelate can help lower lithium level (remember lithium is analogous to potassium in the body).  Andrea discussed a curious case report  about a patient with a lithium level that stayed elevated even though the patient had stopped taking any lithium for multiple days.  It was learned the patient was ingesting their own urine and recycling the lithium in their body.

 

Valproic acid:  Indirectly increases GABA (neuroinhibitory/think benzos), and decreases glutamate (glutamate is neuroexcitatory).   Small volume of distribution like lithium, but it is highly protein bound in the bloodstream.  Consequently, dialysis is usually not useful in standard overdose due to protein binding.  In massive overdose, when protein binding sites are all full and there is still valproate in serum, dialysis can be helpful to remove unbound valproate.

Adverse effects: tremors, hair loss, liver effects.  Not used in kids under two because it can cause a Reyes-like syndrome.

Overdoses can cause hyperammonemia due to depletion of carnitine.   Valproate metabolites interfere with the urea cycle resulting in hyper ammonemia.   Clinical findings include: drowsiness, coma, respiratory depression, and thrombocytopenia.   Massive overdose (>850) can result in hypotension, lactic acidosis, pancreatitis, and bone marrow suppression.   Cerebral edema can result from acute or chronic overdose.

Management: Protect airway, give L-carnitine. There may be some benefit to whole bowel irrigation.  Get serial levels of valproate to document the level is going down.   Elise comment: When should we get a second level?   Andrea:  Check every 2 hours.

Administration    HIPAA Privacy

HIPPA= Health Insurance Portability and Accountability Act

Permitted uses of patient health information:  Can be given to the patient.  Can be used for treatment/payment/operations uses.  Can be used for the Public interest such as abuse reporting.  Limited data sets can be used  for teaching/training, research or performance improvement.

Mistry comment: How do we handle requests by law enforcement officers for patient info?    If the patient may have died by a criminal act, you can release info to the police.   If the patient is in custody, you can release info to police.  If the police are investigating a case, you can give patient info.   Document the officer’s name and badge number and the info you gave the officer.  Give the minimal information needed by the officer. 

You can be terminated from Advocate for posting even generic comments on social media about a case you saw.

When speaking to a patient and their family or friends in the ED, you can give info about the patient to everyone in the room with the patient if the patient consents or in your professional judgment it is appropriate to give to persons whom the patient brought with them.  Probably it is safest to ask the patient if you can speak freely about their medical information with other persons present.   It is stated on the federal HIPAA website that you can use professional judgment about when to discuss patient healthcare info when other persons are present with the patient. Christine comment: What about a 17 yo person who had a drug screen.  Can you tell their parents?   Bottom line yes.  If the patient came with the parent to the ED, the emergency physician can use their best judgment on whether or not to discuss the drug screen with the parent present.  The 17yo is still technically a minor so that has to also be considered. Harwood comment: A bright line decision rule for residents would be at age 18 the patient is considered an adult and probably you should discuss with the patient whether or not you can discuss results with the parent present.

If you feel that a patient is an immediate danger to someone else it is permissible under HIPPA to notify the person at risk.

You can share patient info with providers at other hospitals who are caring for a patient without prior patient authorization.  Photos can be shared between providers in order to care for patients.  Get verbal permission from  the patient before texting a picture to a  consultant.  Document that verbal consent in the chart.  Mistry comment: HIPAA is not meant to get in the way of patient care.     

Don’t save patient data on your computer/laptop/tablet/phone.  If the device has 500 or more patient records and it is lost, that will be considered a major breech in HIPAA.  This requires immediate government notification and a press release.

It is not a HIPAA violation to discuss medical issues with patients who are boarding in the hallway as long as you make reasonable attempts to be discrete (lowered voice, pulled curtains).   Elise comment: Do your best to get patients into a room for sensitive discussions. 

 

 From the HIPAA website:

  

Konicki      Resident Benefits  Selection

 

Conference Notes 6-4-2013

There are 2 images in this document.  If you don't see them, scroll to the bottom and click "Read in Browser".

Conference Notes  6-4-2013

Herrmannh/Kettaneh     Study Guide  Potpourri 

Frostbite:  Treat by warming frostbitten extremity with 39C temperature  water.  Update tetanus shot.  Pain management.  Topical aloe and po ibuprofen may be beneficial and are of low risk.  There is emerging data for intra-arterial TPA.

Thrombolysis - Because frostbite is associated with vascular thrombosis of the affected tissue, administration of intravenous heparin along with either intravenous or intra-arterial tissue plasminogen activator (tPA) has been studied as a potential means of improving outcome [35,36]. A single-center, retrospective review of 32 patients with severe frostbite treated with tPA within 24 hours of injury found the incidence of digital amputation to be 10 percent, compared with 41 percent among patients with comparable injuries not treated with tPA [36].

Evidence in support of thrombolytic treatment of severe frostbite is retrospective and involves only a small number of cases. Nevertheless, treatment options for patients with severe frostbite are limited and outcomes often poor. Therefore, in patients at high risk for life-altering amputation (eg, multiple digits, proximal amputation), without contraindications to the use of tPA, who present within 24 hours of injury, we suggest treatment with intra-arterial tPA. Whenever possible, treatment should be performed at or in consultation with a center experienced in the use of tPA for frostbite.

In the study demonstrating benefit from tPA, a bolus of two to four mg was followed by an infusion started 0.5 to 1 mg/hour given via the femoral or brachial artery [36]. Repeat angiograms were performed every 8 to 12 hours. Treatment continued until perfusion was restored or a time limit of 48 hours was reached.  (Up to Date)

Acetaminophen OD:  Loading dose of IV Nac is 150mg/kg then 50mg/kg for 4 hours then 100mg/kg over 16 hours.  PO loading dose is 140mg/kg followed by 70mg/kg every 4 hours X 17 doses.   The toxic metabolite of APAP is NAPQI.  The 4 hour  level indicating treatment  is 150 on the nomogram.

Kawasaki’s DZ: mucocutaneous lymph node syndrome.  Fever for 5 days/bilateral conjunctivitis with perilimbal sparing is present in 90% of cases/mucositis/rash/indurated edema of hands and feet (last manifestation to develop).   Most common complication is coronary artery aneurysms.  Patients can also get CHF and peripheral arterial occlusion, arthritis, aseptic meningitis.

Bilateral conjunctivitis with perilimbal sparing seen in Kawasaki's (Up to Date)

Edema of hands seen in Kawasaki's  (Up to Date)

Antidote for Cesium-137  is Prussian blue.  It binds Cesium in the GI tract and allows it to be excreted in the feces.   Prussian blue can cause hyperkalemia and bluish discoloration of body fluids.

Oral commissure burn from a child biting electrical cord.  Child will need surgical debridement and possibly reconstruction of the angles of the mouth.   Patients can have severe delayed bleeding at 5-10 days from damage to the labial artery.

Dextrose  for hypoglycemia:  Newborn/ infant gets D10,  Child gets D25,  Older kid or adult gets D50. Divide 50 by the age appropriate D number to calculate the ML/KG dose.     So for example, a newborn: 50 divided by 10 results in 5ML/KG of D10.  Girzadas comment: Important info to know but in resuscitation situation have an app like Pedi Stat on your phone to decrease the risk for error.

Ductal dependant cyanotic heart lesions in infants: Give prostaglandin E1. There is a risk of apnea, hypotension, and tachycardia  when giving prostaglandin E1. So, have airway equipment and resuscitation meds readily available.

Vitamin A toxicity: Intracranial hypertension similar to pseudotumor cerebri.   Adults can have headache, nausea/vomiting, and vision changes.   Infants will have irritability/vomiting/bulging fontanelle.

Fakhouri   Fingertip amputations, Flaps and Other Hand Emergencies

Fingertip Amputations:  Save the tip.  Wrap it in moist gauze, put it in a plastic bag, then place it on ice.   The thumb is the most important digit of the hand; It is worth ½ the hand.   Don’t rongeur anything on any finger.  If bone is exposed,  just wash and cover with xeroform dressing followed by dry dressing.  Needs hand evaluation within 24 hours.   The thumb is critical, it has to be rescued/reimplanted at all costs.   For the other fingers, if the amputation is at the PIP or proximal, the patient will probably loose the finger.  Reimplantations at the PIP joint  or proximal have very limited function.   Consider admitting any injury more severe than a tip avulsion.  If you see macerated or mangled soft tissue, it likely is not re-implantable.   Save every amputated digit until the surgeon evaluates the patient.

Nailbed  Injuries:  Fix the nailbed with absorbable suture.  Trephinate the nail inside to out, Replace the nail into the nailfold and suture in place.   Nail needs to be sutured in place for 2 weeks.  Place suture through nail and then through the superficial eponychium proximally and through the nail and the soft tissue distally.

Scapholunate dissociation:  3mm or more between the scaphoid and lunate.  These patients if untreated will develop arthritis in the long term.

Dorsal dislocations of PIP splint in flexion after reduction.   Reduction is best obtained with distal traction and applying pressure with the physician’s thumb on the volar aspect of the distal portion of the proximal phalanx.  Again, splint reduced finger in flexion at the PIP joint.

Erickson/Salzman    Trauma Lecture

You need to intubate with an 8.0 tube for trauma patients in case the patient needs to have bronchoscopy.

Salzman comment: Aspiration is a disaster.  Consider cricoid pressure to prevent passive aspiration.

Although Sellick's maneuver may reduce gastric insufflation during bag-mask ventilation, evidence that cricoid pressure reduces the incidence of aspiration of gastric contents is scant and consists primarily of observational clinical studies and experimental data [23]. Several studies suggest it may contribute to airway obstruction and difficulty intubating in some cases, even when a video laryngoscope is used [23-27]. Until more definitive literature is published, we suggest that the use of Sellick's maneuver during RSI and bag-mask ventilation be considered optional.(Up to Date Reference)

A systematic review of cricoid pressure studies noted the following [23]:

  • The literature provides evidence both for the success and failure of cricioid pressure to prevent aspiration
  • Cricioid pressure is often used improperly
  • Cricoid pressure may impair the function of the lower esophogeal sphincter 

Possible risks from cricoid pressure include movement of unstable cervical spine fractures and esophageal injury.   (Up to Date Reference/Ellis DY et al. Cricoid Pressure in Emergency Department Rapid Sequence Tracheal Intubations: A Risk-Benefit Analysis Annals of EM  Dec 2007)

Girzadas comment: Cricioid pressure carries some risk of esophageal rupture if used in a patient with active emesis.

Salzman comment: Criteria for OR based on Chest Tube output: 1500ml initial output.   250ml/hour output over 4 hours.

Salzman recommended large caliber chest tubes for trauma 36-40F.   Harwood comment: Smaller patients’ intercostal spaces may only accommodate 32F tubes.   Both agreed that it was critical to make sure the tube was well secured to the chest wall.

Discussion of pain medication for rib fractures:  There was consensus between Salzman and ER faculty that aggressive pain medication was warranted in trauma patients.  The risk of respiratory depression in trauma patients due to opioids is low.  Dr. Salzman made note of an exception to this rule regarding elderly trauma patients who may be more likely to have respiratory depression.

Case vignette: 25yo male with traumatic amputation at distal tib-fib.   Salzman’s point: Don’t focus on the mangled  extremity.  Stick to your primary survey initially prior to dealing with the more visually graphic extremity injury.   Similar issues apply to other emotionally traumatic situations like injured children/pregnant women/other visually stunning injuries.

Barounis   Last Lecture

Case: 6wk old male.  Vomiting.  Eval in ED for pyloric stenosis.  U/S showed hepato-splenomegaly.  MRI of abdomen showed diffuse neuroblastoma.    Dave cared for the patient in the ICU and got to know the family.   Every time the patient left the ICU he seemed to have some complication such as ET tube dislodgement.  Parents became wary of procedures.   Dave became the family’s trusted clinician.  They wanted him there anytime a procedure was done.   Dave learned the importance of earning the trust of the patients and families that come to the ER.

Passion in the ER/for the ER: latin pascio=suffering.   You give up a lot to become an ER doc.   Create your own mission statement to kindle your passion for whatever you do.

If you want to understand someone, you have to consider things from their point of view.  Climb into their skin and walk around in it.

As a leader, Better than telling others what to do, Inspire others to do what needs to be done.

Don’t be a problem identifier, be a problem solver.

All eyes are on you.  People watch you as a leader and follow your example.    Be a positive example for people.

Be decisive in the ED.  Even if you are wrong, it’s better than being indecisive.

Be the change you want to see.

Big rocks in the jar analogy.  Schedule time for the things most important to you first.

Senior Pearls Lecture    Excellent Humorous Advice from Graduating EM3's

Not intended for publication on the website other than Ted Toerne's Advice: The fastest way to discharge a patient from the ED is to "Click the Discharge Button on the EMR"

Conference Notes 5-28-2013

There are images in this document.  If you don’t see them scroll to the bottom and click  “read in browser”

Please consider donating to our EM Foundation to benefit resident education/development.  Thanks for your consideration/generosity.
/em-foundation/

Conference Notes

Lovell/Cash     Resident as Teacher Workshop

What makes a good clinical teacher: clinical knowledge, positive relationships, supportive learning environment, communication, enthusiasm.   As you can see, most of these are non-cognitive traits/skills.

Good EM teaching principles:  The teacher takes time to teach, gives appropriate feedback,  tailors teaching to learner, using teachable moments effectively,  positive attitude.

Barriers to teaching: time management, clinical load, interruptions, loud environment,  personal  motivation, interpersonal issues,  dealing with “non-exciting cases”,  some learners have very limited knowledge base and finding  their knowledge gaps and working with their gaps can be difficult,  and physical space limitations.

Research has shown that 70% of medical students feel their supervising residents had a significant role in their learning.  They felt that about 30% of their learning came from resident teaching.  So the teaching residents do with their students is very important.

Traditional teaching model:  Teacher acts as an expert consultant, the model is centered around the teacher, teacher transmits what they know to the students.   This model is mostly a one way street.   Learners are passive. Learners are uncomfortable exposing their knowledge gaps.

Learner centered model:  There is back and forth learning between teacher and student as well as between students.   The teacher takes time to find knowledge  gaps of learners.   Learners demonstrate reasoning and problem solving to teacher.  Teacher gives quick, relevant teaching points in a non-threatening manner.   Focus is more on learners’ reasoning.  Learners are encouraged to ask questions.  Learners are active in this process.

One Minute Preceptor Technique:  6 steps (or microskills):  1.Get a commitment (teacher asks specific questions about a case to get the learner to invest themselves such as “ what do you think is going on with this patient?”),2. Ask learner for supporting evidence (teacher is looking for knowledge gaps by asking probing questions such as  “What clinical findings suggest endocarditis?”), 3.teach a general principle (give the learner a short pearl or take home point), 4.reinforce what was done well (start with positive feedback on specific aspect of the students clinical reasoning or exam finding), 5.correct mistakes (give corrections in a positive manner) , identify the next learning steps (encourage specific further reading) .     Ryan comment: Thinking out loud with the student is very useful.  Even just talking about how your are deciding to see which patient first is teaching the student.   Mistry comment: Some of the basic mundane tasks we do are still very interesting and novel for students.

SPIT  Technique:  Serious   Probable   Interesting    Treatable.  This is a quick method to develop a differential diagnosis.  Have the student give diagnoses for each component of the SPIT acronym for a patient case. You can do it before you see the patient and after you see the patient.   For example in a patient with chest pain: Serious would be aortic dissection or ACS, Probable would be chest wall pain or GERD, Interesting would be shingles or pneumothorax or pneumomediastinum, treatable would be ACS/aortic dissection/GERD/Chest wall pain/Pneumothorax.

Harwood comment: To give feedback, you can ask the learner what they think they can do better.   That will usually start a useful discussion and give the teacher a non-confrontational way to discuss ways to improve.  Always use the PNP sandwich.   Positive-Negative-Positive.    Tell the learner something positive about their performance, then the specific area of potential improvement, then another positive to build them back up.    Mistry comment: Ask student what was the most interesting thing you saw/learned today and then follow up on that with some learning points or suggested reading.   Elise comment: You can take that one step further by then telling the student to teach that learning point to someone else.    Feedback is important and most students feel they don’t get enough of it. To improve this situation, the teacher should make an announcement to the learner “I am now going to give you some feedback”.   Gottesman comment: I tell the student at the start of the shift that I am going to give you feedback at the end of the shift.  This prepares the student for that feedback and also encourages them to be invested in the shift all day and to expect feedback.   Elise comment: you can give feedback during the shift as well; it doesn’t have to be at the end of the shift.  Harwood comment: The teacher’s self critique of their own teaching or clinical care is a very effective teaching technique to students. Harwood comment: If you want to teach on a “boring case,” you can ask questions about the case with one variable changed.  Example:  abdominal pain + pregnant or +gi bleeding or +afib or +previous surgery.  You can ask the learner how they would manage the case with these variables. 

Walchuck   Trauma Myths

Myth 1.Resuscitate your patient to normal vitals:  Truth:   In penetrating trauma keep MAP around 65/heart rate in range of 60-100.    Barounis comment: This is tough, it your instinct to give a lot of saline to get MAP higher.   Resuscitate with blood and plasma.  Use lower dose of sedative (avoids worsening hypotension) and higher dose of paralytic (poor perfusion in hypotensive patient).   Ketamine can be used.  It doesn’t increase ICP.  Always Be prepared for patient to arrest during intubation.  These hypotensive patients are very unstable.  In blunt trauma and closed head injuries, keep MAP up to 105. In non-hemorrhaging  injured patients, the brain does better at a higher MAP

Myth 2.  In the setting of pelvic trauma, blood at meatus is absolute contraindication to foley placement.  Truth:  It is reasonable to attempt one gentle pass of a foley catheter.   Steve noted there was data to support this approach and that he spoke with the Trauma attendings who have anecdotal success with this method.  Girzadas comment: I was hesitant to accept the approach of placing a foley through a bloody meatus without a retrograde urethrogram, but there is a statement from the European Urologic Society 2013 Trauma Guidelines supporting 1 gentle attempt at foley placement prior to urethrogram although they do hedge a bit:

 Blood at the meatus is present in 37-93% of patients with posterior urethral injury, and in at least 75% of patients with anterior urethral trauma (31,32). When urethral trauma is suspected an attempt of urethral catheterisation should be carried out by experienced hands and the balloon of the Foley catheter should only be inflated if clear urine flows out. It is extremely unlikely that gentle passage of a urethral catheter will do any additional damage (33,34). Although it has been suggested that passing a catheter may convert a partial tear into one that is complete (35), there are no convincing data indicating that there is a higher rate of infection or urethral stricture after a single attempt at catheterisation (1). However, the most prudent approach is to carry out urethrography prior to an attempted catheterisation. In an unstable patient, an attempt to pass a urethral catheter should be performed, but if there is any difficulty, a suprapubic catheter should be placed using ultrasound guidance and a retrograde urethrogram must be performed once the patient has been stabilised.

Myth 3. Pulse exam rules out extremity vascular injury. Truth:  You have to look for hard signs of vascular injury in penetrating trauma: hemorrhage/pulsatile hematoma/bruit or thrill/absent pulse/signs of ischemia.  If any hard sign is present in association with a penetrating injury, the positive predictive value is virtually 100%, so patient goes to OR.   Hard signs are less reliable in blunt extremity trauma.  In patients with a penetrating extremity injury and soft signs only such as a pulse difference, do an ABI.  A normal ABI is >0.9. If the ABI is normal then there is no need to image and patient can be observed and have serial ABI’s.  If <0.9 then further imaging like a CTA is indicated.

Myth 4.  Pelvic exam not necessary in trauma.  Truth: Speculum exam can be very important in pelvic fractures to be sure there is no vaginal laceration.  Also water skiing or jet skiing injuries can cause serious vaginal tears including intra-peritoneal free air.

Carlson          Electrocardiography in Overdose

QRS is affected by na channel blockers like tricyclic antidepressants/cocaine/tegretal/Benadryl

QT interval prolonged by K channel blockers like methadone/macrolides/antipsychotics/ibutilide and amiodarone

ST segment is affected by cocaine/sympathomimetics/ergotamine/cellular asphyxiants like cyanide or CO.   This is probably most common in CO poisoning and the recommendation is to go to hyperbaric not cath lab.  Most patients who go to cath lab for CO related ST elevation have clean coronary arteries .

U waves are caused by Lithium/methylxanthines/hypokalemia/hypomagnesemia

Case 1.  60yo female, dizzy with ekg showing a lot of deep t wave inversions diffusely looking like subendocardial ischemia.   Rhythm looks like afib.   Pt had digoxin toxicity.  Ekg with dig can have “hockey stick shaped”  st changes.   Dogoxin inhibits na pump with increases intracellular calcium which in turn increases contractility force.   Digoxin also has vagotonic/anti-adrenergic  actions.

Case 2. 70yo female who is somnolent.  EKG is aflutter and ventricular bradycardia.    Pt has classic ekg (atrial tachycardia with block) for  digoxin toxicity. 

Virtually every dysrythmia has been reported with digoxin toxicity.  Bi-directional V-tach is considered pathognomonic for digoxin toxicity. 

Bidirectional Ventricular Tachycardia

Digoxin also has GI/mental status/vision effects.   Increasing potassium levels are an indicator of the severity of digoxin overdose.  For patients with V-tach or V-fib.  Follow normal ACLS guidelines.   The fear of giving calcium in the digoxin toxic patient is overblown.  Don’t wait to get digoxin level if patient needs calcium for severe hyperkalemia.  Give digibind for K>5.5/ventricular dysrhythmias/slow rate not responding to atropine/combo digoxin and CCC overdose/Digoxin level>10 or ingestion >10mg.  Harwood comment: give digibind in hypotensive patients.   Digibind takes 20 minutes to work.   Renal dysfunction is a common factor in digoxin toxicity.

Case 3. 30yo male with syncope at work. Hx of bipolar dz.  EKG with long QT.  Pt later went into torsades.  Pt was on ziprasidone and was put on erythromycin.  This drug combo caused prolonged QT.   Elise comment: be careful when considering ibutilide in alcoholic patients.  Alcoholic patients tend to be hypomagnesemic and may have prolonged QT interval.  Don’t give ibutilide in patients with a long QT. Discussion about zithromax and risk of prolonged QT among attendings:   General consensus was that zithromax was safer than other macrolides and was not a risk to patients.  Harwood comment: Original study on this issue was a population based study in medicare patients showing some increased risk of death.  Later study in Europe of younger patients showed no increased risk of death due to Zithromax.

Case 4.  SVT due to caffeine.   Methylxanthines include caffeine and theophylline.  Methylxanthines will counteract the effects of adenosine. You may have to use higher doses of adenosine.  Sympathomimetics and herbal supplements (bitter orange) can cause SVT. Weight loss supplements commonly contain caffeine like substances

Case 5.  19 yo patient took clonazepam and propranolol.  EKG showed sinus brady with brugada pattern.  Brugada pattern can develop from various toxins.  If brugada pattern develops from toxin, the patient does not need an AICD. The patient does not have a chronic cardiac conduction abnormality, they are suffering from a transient toxic effect.   TCA’s and cocaine can do this.

Case 6.  TCA overdose.   EKG shows TCA pattern is prolonged QRS, Tall terminal R wave in AVR. 

TCA EKG 

TCA’s are dirty drugs with multiple pathophysiologic mechanisms.  These mechanisms include Antihistamine/alpha blocker/na channel blocker/k channel blocker/GABA blocker.   Treat seizures with benzos.  Second line anti-seizure med  is phenobarb.  Avoid phenytoin in TCA overdose.  Avoid 1a and 1c class antiarrythmics and beta and calcium blockers.  Main antidote for cardiac effects is sodium bicarb.  Sodium bicarb doesn’t really work on neuro effects like seizures.   Shoot for a serum ph of 7.5-7.55.  Give bicarb for QRS>100/severe acidosis/hypotension/cardiac arrest.    6 hour observation period will declare the “sick” vs “not sick “ patient.  Harwood comment: TCA’s can have a “ Catastrophic demise”.

Case 7.  19yo took overdose of valium.  EKG shows complete heart block.   You can google the imprint code on a pill to identify what drug it is.   The ER docs found that the patient didn’t actually take valium but mistakenly took verapamil.   Treat CCB’s with calcium/atropine/glucagon/high dose insulin/intralipid

 

Conference Notes 5-21-2013

There are 6 images in this document.  If you don't see them, scroll to the bottom of the page and click "view in browser".

Tekwani       Study Guide

Inflamatory Bowel disease: Peak incidence is between ages 20-30.  Patients will have abdominal tenderness.  May have blood in stool.

Best management of uncomplicated diverticulitis: no ct scan and treat with oral antibiotics.    Complicated diverticulitis: has phlegmon, perforation, stricture, abscess,etc, Ct is recommended if diverticulitis is suspected to be complicated by any of these

Ascending cholangitis due to obstructing stone: Treat with ERCP, IV fluids, IV broad spectrum antibiotics

If you find an anal fissure out of the midline, pt needs further consideration  of HIV, cancer, crohn’s, syphilis, or foreign body insertion.    Midline anal fissure is the most common cause of painful rectal bleeding.  90% are posterior and 10% are anterior.  Midline anal fissures are due most commonly to hard stool.

Amebic liver abscess:  entamoeba histolytica. Tx with hi dose metronidazole, think of it in patients from AZ or mexico or other endemic areas.  Develops from eating food contamintated with feces.   Pt’s present clinically about 8 weeks or more after travel.  They have RUQ pain and fever.

 

Amdebic liver abscess

If patient has conjugated hyperbilirubinemia think obstruction of biliary tree: stones/atresia/sclerosing cholangitis.   If ptatient has unconjugated hyperbilirubinemia, think of things that cause red cell destruction: hemolysis/sickle cell disease

Intersphinteric abscess develops from an anal gland obstruction and is the most common cause of anal fistula.

 

RLQ tenderness has the highest Liklihood Ratio of clinical findings for the diagnosis of appendicitis.

Work up for appendicitis in pregnancy: CT of abdomen/pelvis in the first trimester doubles the risk of childhood cancer in the fetus.    MRI is an option but don’t give gadolinium in the first trimester. Most common option is to consult surgery for serial exams.

Acute ETOH hepatitis: enlarged liver and AST,ALT both elevated with a AST/ALT ratio greater than 2.

Worldwide, the most common cause of acute liver failure is APAP overdose.   Most common cause of chronic liver failure is hepatitis.

Volvulus: 75% are sigmoid.  Sigmoid are seen in elederly/bedridden patients. Can treat sigmoid volvulus with colonoscopy.  25% of  volvuli are cecal.  Seen in marathon runners/young patients. Treatment is surgical.

Initial treatment for toxic megacolon is fluids, antibiotics and steroids.

Hepatitis A never causes chronic hepatitis

IBS is more common in women, pain relieved by defecation. It is not a psychiatric disease.

Most common cause of large bowel obstruction is neoplasm.

Most common cause of massive lower GI bleeding is diverticulosis.   Diverticulosis also most common cause of lower GI bleeding in general.

Lipase is more specific than amylase for pancreatitis. Both have similar sensitivities.

Toxic megacolon more common in UC.   Perianal complications more common in Crohn’s disease

Schroeder       Visual Diagnosis

Clinical diagnosis of anemia in dark skinned individuals: check the conjunctiva and palms.   Palms will be more pale and less pink in the anemic, dark skinned individual. 

 Febbo question:  Do we need to get written consent from patients to send a picture of an injury/illness to a consultant with our phones.    Mistry/E. Kulstad response: No.  Just get verbal consent from patient prior to sending photo to consultant.    Be careful to limit any identifying info.   Elise comment: Only do this for patient care.   Do not send any images just because it is a cool or interesting picture.  You can end up in a lot of trouble.  Girzadas comment: Document pt’s verbal consent in the medical record.

Pitariasis rosea: Believed to be a viral illness.  Rash lasts about 3 months.  Kelly comment: Latest research is mixed on contagiousness.    Rash is benign.   Benadryl can be helpful for itching.  Sun exposure helps relieve rash.

Gutate psoriasis:  Small red patches with scaley surface. Gutate means” drop” and patches are raindrop size.    This rash can follow strep infection by 2 weeks in genetically susceptible patients.   Sun improves this rash.  

Internal/external hordeolums can be treated with warm compresses and topical antibiotics.

Cradle cap: Treat with baby oil and combing out the flakes.  If more severe, you can use lotrisone topically.  Rash is greasy/oily appearing rash.   As a contrast, eczema has a dryer appearance.

Numular eczema: coin shaped eczema in middle aged persons.   This is more common in atopic persons or persons with hx of asthma.

Strep throat/scarlet fever: strawberry tongue with papillae and palatal petechiae.  Scarlet fever has sandpaper appearance.    Discussion about bicillinLA: Consensus that treatment with bicilin LA was totally acceptable if parents/patient prefer or if you are concerned about patient compliance.

Infected ear pits usually need surgical drainage.  Can start with oral antibiotics and arrange surgery with ENT.  Cover MRSA.  Elise comment: don’t pop or drain in ED.  It will be ineffective.   There is some possible association between ear pits and renal malformation.  There was consensus in the room that if you diagnose this, you don’t need to get a renal ultrasound on the patient.

Potts puffy tumor has mid forehead swelling.  Admit for IV antibiotics. There is risk of intracranial extension.  Infection most commonly involves anterior wall of frontal sinus.  If toxic appearing get CT head to look for intracranial extension.  If not toxic appearing, ok to admit with iv antibiotics and plan for mri of brain.

Potts puffy tumor

SSSS usually causes kids to be more uncomfortable/febrile/toxic  vs impetigo where the kid is usually nontoxic/happy

Kerion: boggy inflammatory mass on scalp due to tinea capitus.  Requires oral antifungals.  Bill just starts griseofulvin and does not draw baseline labs.  Diflucan is second line treatment.  Don’t excise or drain.   

Kawasaki’s conjunctivitis has perilimbic sparing with no drainage.  Classic mimics of kawasaki’s are scarlet fever and adenovirus.

Sublingual swelling can be from a mucocoele or rannula.   Requires elective excision.

Alopecia areata is well circumscribed hair loss.

Erythema multiforme: classically due to herpes, mycoplasma, medications.   Rash may have target appearance.  Elise  comment: We see this very commonly.  It is rare to progress to Steven Johnsons.  Don’t over-react to this.  Bill comment: if pt is on a sulfa drug stop it.  Consider admission or good instructions to parents to watch for oral lesions or skin sloughing.   Consider acyclovir for patients with cold sore or exposure to child with cold sore because herpes infection is more likely with this clinical scenario.  Bill does not use steroids for this rash.

Ryan   Med Student Elective Update

Heller   Pediatric Abdominal Radiology

Consult your imaging consultants if you have uncertainty about the best imaging study to answer a clinical question.

Neonatal bowel obstruction:  Clinical exam cannot differentiate between high and low types of obstruction.   Bilious emesis is a marker of malrotation.  However, bilious emesis in a child less than 36 hours old doesn’t always mean malrotation. Begin imaging with a plain abdominal xray in the child under 36 hours long.    Double bubble sign is almost always duodenal atresia and requires surgery

Double bubble sign

 Upper obstruction can be due to Malrotation that  lead to midgut volvulus.  Bilious emesis in the newborn who has been normal for a few days should be considered midgut volvulus until proven otherwise.   Elise comment: no need to do a plain xray in the previously normal newborn with bilious emesis.  This child has likely midgut volvulus and should go right to upper GI.   Non-bilious emesis in 4-8 week child, think pyloric stneosis.   In a normal child you can’t even find the pyloris on u/s.  Harwood comment: what do you do with kids with borderline U/S findings for pyloric stenosis?  Heller reply: We re-image in 48 hours.  If child dehydrated or not feeding, then admit.  If hydrating OK then child can go home and get second U/S as outpatient.

Low neonatal obstruction usually shows numerous gas filled bowel loops on KUB.   Low obstructive causes: imperforate anus, hirschprungs, small left colon, meconium ileus, ileal atresia.   Work up for low obstructions is barium enema.    Meconium ileus is thick meconium obstructing the bowel.  Associated with CF. Hirschprungs is due to an absence of ganglion cells in rectum. Pt’s have a low rectal/sgmoid ratio.   Basically the rectum is narrower than sigmoid.  Associated with downs syndrome.

 

Low Obstruction from Hirschprung's Disease

Adhesions/appy/inguinal hernia/intussusceptions/malrotation/meckel’s    AAIIMM mnemonic for SBO.

Intussusception: 90% are ileo-colic.   90% are idiopathic.    Ideopathic means lymphoid hyperplasia is thought to be causing intussusception.   Sweet spot for age for intussusceptions is 6mo to 2 years.   More common in winter/spring.   Initially patients will have crampy abdominal pain.  If this progresses without identification over a couple of days then kids become lethargic.   Start with KUB.  Second step is U/S.  U/S should be able to diagnose or exclude the disease. Heller felt that U/S is a good test for intussusception.  Treat with enema reduction (air or liquid contrast).  Younger radiologists prefer air contrast enemas.  Barium for enema is risky because if there is a perforation it causes peritonitis.   Gatrograffen is also risky because it is hyperosmoler.   If there is a perforation it can cause life-threatening electrolyte abnormalities.   Radiologists dilute gastrograffen 5:1 with water to reduce osmolality.   Air contrast enema does not carry an increased risk of perforation.    You have to have surgery on alert prior to doing a barium enema.

Appendicitis:  Start with U/S.   2nd test is an appendicitis protocol CT.  This is a pelvic CT with IV and enteric (oral or rectal)  contrast.   Heller’s comment is if you are going to expose the child to radiation,  take your best shot with both IV and enteric contrast.  Herrmann comment: sometimes this study can miss the appendix.  Heller reply:  You see the appendix 99% of the time with this study.  Non-visualization of the appendix and secondary signs of inflammation is reliable evidence that makes appendicitis much less likely.  Most common missed diagnosis with limited appendicitis protocol is pneumonia.

Radiation safety:  Lifetime Risk of fatal malignancy from CT is approximately 1:1000.

Conerence Notes 5-14-2013

Conference Notes  5-14-2013  There are images in this document. If you don't see them, scroll to the bottom of the page and click "view in browser"

Please consider donating to our EM Foundation to benefit resident education/development.  Thanks for your consideration/generosity.
/em-foundation/

 

Kessen/Collins          Oral Boards

Case 1.   28yo female with syncope due to ruptured ectopic pregnancy  Critical Actions:  Diagnose ectopic and identify intraperitoneal blood with FAST exam. Resuscitate with IV Crystalloid and give emergent PRBC transfusion when crystalloid fails to improve vitals.   Give Rhogam for rh neg mom.    Get OB-Gyne to take patient to OR emergently.

Case2.  27 day old male with  fever and “not acting right”   CSF shows signs of meningitis. Critical Actions: Septic work up including LP and stool culture (history of diarrhea), IV boluses, IV antibiotics (ampicillin and cefotaxime), check for signs of abuse. 

Case3.  38yo male with reported suicidal ideation.  Unknown ingestion later identified as APAP overdose.  Patient also had access to a firearm.  Critical Actions: Get important history from parents and girlfriend. Give NAC  for APAP overdose.   Search patient for any weapon.  Dave’s comment: there was a recent patient with suicidal ideation in the ED with a knife hidden in his sock.  Dosing with oral NAC is 140mg/kg PO first dose. Followed by 70mg/kg doses PO q4 hours for 17 doses.  No adjustment of dosing is needed if oral charcoal is given.  There are shortened oral courses of NAC in the literature for Certain clinical situations.  You should consult a toxicologist before stopping oral treatment before the 17th dose.

The approved 20 hour IV dosing regime is complicated and is performed as follows:

Administer an initial loading dose of 150 mg/kg IV over 15 to 60 minutes (we recommend 60 minutes).

Next, administer a 4 hour infusion at 12.5 mg/kg per hour IV (ie, total of 50 mg/kg over 4 hours).

Finally, administer a 16 hour infusion at 6.25 mg/kg per hour IV (ie, total of 100 mg/kg over 16 hours).   (Up to Date Heard, K et al)

 

Kettaneh/Herrmann                   STEMI Conference

Case 1. 78 you female.  PMH: seizure ,hypertension and high cholesterol.  Presents with syncopal episode/head injury with no chest pain but pt has nausea.     First EKG was not clear cut.  Dr. Silverman  noted somewhat diffuse st elevation with no reciprocal changes.  2nd EKG shows no evolution. 

 Dr. Mistry comment: Documentation is important in this type of case “PCI delayed due to additional diagnostic testing needed.”   Pt went to CT head. CT showed no acute bleed. Troponin was elevated.  Pt diagnosed as NSTEMI.  Dr. Silverman comment: A stat Echo in the ED would have been helpful to identify a focal wall motion abnormality.   At night the tech can take a clip of the image on their smart phone and transmit it to the cardiologist.  Pt was in ED overnight and was hypotensive.  3rd EKG in AM shows evolution and ischemic changes.

Patient was taken to cath lab.   Cath lab shows no coronary stenosis.  Diagnosis was Takotsubo’s myocarditis.   Silverman comment: Takotsubo’s is particularly difficult to diagnosis and is becoming more common.  Only way to diagnose is cath.

Common causes of false-positive STEMI alerts: early repol, myocarditis/pericarditis, takotsubo cardiomyopathy, coronary spasm.  Risks for false positive activations: no chest pain, no reciprocal change, less than 3 cardiovascular risk factors, symptom duration >6 hours.

Takotsubo’s CM: apical ballooning syndrome, presents with chest pain/syncope/dyspnea, ST elevation and elevated troponin but pt has no obstructive coronary lesions.  More common in post-menopausal females.

Case 2.   55yo male with chest pain.  First EKG with signs of posterior STEMI and subtle inferior ST elevation.  

Dr. Silverman comment: Judgment call on this first ekg but with good story, would call a STEMI activation.  2nd EKG 20 minutes later shows Inf-Post EKG.  

Dr. Silverman comment: 20 minutes is a long time to wait to get a second EKG when the first is concerning.  Posterior leads showed posterior STEMi.  Pt had a circumflex injury on cath.   Circumflex lesions are the most common cause of posterior MI.  Posterior MI’s are often associated with inferior MI’s.  Look for horizontal ST depression with a prominent R wave in the anterior leads.   WPW can also cause prominent anterior R waves.  When you do posterior leads, place the V4-6 leads on the left back with the middle lead placed on the inferior tip of the scapula and the other two leads placed symmetrically  around the middle lead.  These three leads become V7-V9.

Case 3.  59yo male with chest pain.  Pt staes he had 7 previous MI’s.  First EKG with inferior ST depression and mild high lateral ST elevation.  Anterior t waves are inverted as well. Dr. Patel comment: first EKG does not meet strict STEMI criteria but is concerning.  Pt developed more chest pain in ED and 2nd EKG showed Acute High lateral AMI. STEMI was called. 

Cath showed occlusion of ramus intermedius artery.   The Ramus intermedius is a coronary anomaly present in 35% of patients.  It is a trifurcation of left main coronary artery. Can supply anterior, lateral or high lateral heart regions.   Again documentation should read:  Delay in PCI due to additional diagnostic testing needed.   Other exclusionary statements include:   PCI delayed due to st elevation in only one lead.  Or PCI delayed due to initial ekg with less than 1mm of ST elevation.   Don’t use the words “mild ST elevation” that does not kick out the case from the CMS bundle.   Mila comment: We have templated phrases in FirstNet.   Dr. Patel comment:  If ST elevation is decreasing from V1-V6 think PE.  Most AMI’s with antero-lat st elevation have increasing elevation from V1-6.

Possible example of decreasing amplitude ST elevation anteriorly c/w pe

Chastain     M and M

49yo male with refractory V-Fib due to respiratory arrest from Influenza Pneumonia

Airway:  LMA can be an effective bridge device for the difficult airway

Anchoring bias: Tendancy to rely on initial information when making decisions.  A Way to reduce this bias is to be aware of it.   Briefly consider alternate/opposite possibility.   Chintan comment: good team work helps to mitigate this bias.

Mistry and Girzadas comments: the Paradigm of leadership is not always the authoritarian dogmatic approach.  A better model is a leader with a global view of the resuscitation who is open to team members suggestions and concerns.

EZIO can be life-saving for  vascular access in the crashing patient.

Setting up zoll pads can be time consuming.   Erik comment: don’t forget the paddles.  They are the fastest option for defibrillating the crashing patient.

Case2.  2yo with severe angioedema.  Pt got SubQ epi prehospital.  Airway with mild stridor and tongue is markedly swollen.  O2 sat is 96% on nasal cannula O2.  IM Epi given, IV steroids given in the ED.  CXR shows pneumomediastinum with diffuse subQ air and bilateral apical pneumos.  Pt has repeated emesis.  

Awake intubation with Ketamine and atropine.  Glidescope was used and anesthesia back up was present.   Things learned: Pediatric bougie does not fit down a 4.0 ET tube.   Pediatric Glidescope does not come with a rigid stylet.

Discussion among faculty: What would be best airway approach between fiberoptic nasotracheal intubation by anesthesia or glidescope intubation by EM in this patient.  

Pt decompensated during airway management attempts.  Airway had to be obtained with surgical tracheotomy in ED.  Bilateral needle thoracostomies and chest tubes were placed for pneumothoraces.

The “angioedema” patient was experiencing was really severe subcutaneous air from chest trauma.  Patient was discovered to have been physically abused.

Pneumomediastinum:  3 main sources esophageal, alveolar, or trauma.  Air can extend into the submandibular space, the retropharyngeal space.   Spinacher sign with pediatric pneumomediastinum is due to displacement of thymus to right side of chest due to air pressure.

 

spinnaker sign

Peds Airway Pearls: Due to a decreased functional residual capacity, kids desaturate quite quickly compared to healthy adults.    You don’t need to put blade in the oropharynx deeply initially.  Just initially go to the base of the tongue and assess your position. Remember  passive oxygenation at 5L per minute.

Difficult cases can affect physicians emotionally and phsycially.  Group discussion of the emotional toll of difficult cases on physicians.  Girzadas comment: ER docs are the people with the guts to "be there" for patients, even the super difficult cases.  Sometimes you are just "that guy" who is there caring for a patient whom no one can save but you still feel like you should have.   Mistry comment: All you can do is your very best.  We can't always control the outcome.  Willison comment: We are compared to pilots with regard to safety, but our job is tougher.  We can't decide not to take a case because it is too hi risk.  Pilots only fly one plane while we manage multiple patients at a time.  If one of our patients "crashes" we can't go off line and stop working. 

 

Conference Notes 5-7-2013

Conference Notes 5-7-2013  There are pictues in this document. If you don't see them scroll to the bottom and click view in browser.

Please consider donating to our EM Foundation to benefit resident education/development.  Thanks for your consideration/generosity.
/em-foundation/

A special shout out to all the Alumni who lectured and visited conference today.   It was a blast to see everyone!   Thank you to the lecturers and thanks to everyone who contributed to the outstanding discussion. These notes won’t do it justice.

Erickson/Tekwani    Oral Boards

Case 1  Neonatal Chlamydial pneumonia and conjunctivitis  Critical actions: Diagnose with conjunctival/nasopharyngeal culture. PCR not yet approved for infants.    Treat for Chlamydia pneumonia  with oral erythro for 14 days.  You can’t just treat with topical ophthalmic erythromycin.

Case2  Torsed appendix testes    Critical actions: pain control, rule out torsion, counsel parent that this is a benign process.    Pain can be similar to torsion.  Blue dot sign.  Nl cremasteric reflex.

Case3  Acute chest syndrome Critical actions:  IV antibiotics, consult hematology and start exchange transfusion.  Causes: infectious (viral  and bacterial etiologies)/pulmonary infarct/fat emboli.     Diagnose based on new infiltrate with fever and chest pain.

Elise comment:  You don’t need to treat torsed appendix testes with antibiotics.  Some GU faculty request antibiotics to be given but there is no good evidence for antibiotics for torsed appendix testes.

Harwood/Elise comments:  For the boards, no one would criticize you for working  up the infant with suspected chlamydial pneumonia with cultures/urine/ lp for possible sepsis. 

Dan Miller  Class 2008     Real Life 5 years out as an Attending     Case F/U’s

Case 1

16 yo male with sob.   Pt had history of muscular dystrophy and a peg tube.   HR=144, BP=43/22, T=35.2 R=44.   No IV access.   There was asymmetry of pupils.   Abdomen was distended and tense.   Med student gets an EJ line. Dan said if he had a redo he would have placed an IO line.    Child was bolused up with normal saline.   Antibiotics started. Vanco and zosyn was given.  Zosyn was given first to cover gram negatives primarily.   He was concerned that Gram negatives would be more likely to cause rapid demise.

Intubation with rocuronium and ketamine.   Don’t use succinylcholine in kids with muscular dystrophy because you can cause severe hyperkalemia.      Pt was difficult to bag.  Abdomen was very distended.  Very difficult to place ng or og tube.  Xrays show alot of gas in abdomen and free air with tension pneumoperitoneum.   Peds surgery attending percutaneously needle decompressed the abdomen.  Vital signs improved and bagging became much more easy.   Pt went to surgery and found to have fecal impaction and resulting bowel perforation.

Tension pneumoperitoneum:  can occur after aggressive bagging, ERCP, volvulous, and others.  Presents with hypotension, respiratory distress, and abdominal distension.  Treatment is with 14g needle decompression (use paracentesis needle).   Stick needle in anterior abdominal wall away from liver.  Put pt in right lateral decubitus postion and stick needle lateral to rectus abdominus muscle.  Pt will either need serial abdominal exams or go to the OR following needle decompression of abdomen.

Asmita comment:  You could also decompress bowel through peg tube site.

Tension Pneumoperitoneum

Case 2

18yo college freshman presents with a seizure.  Blood glucose level of 40.   Pt denied any pmh.  Hadn’t eaten for 2 days due to sore throat.   HR=112,  BP=106/77.   Exam showed a severe pharyngitis.  Rapid strep test was positive.  UCG was negative.   After feeding patient, her blood sugar trended back down to the 50’s.  Pt started on D5.45, admitted.  Signed out.    Pt arrested in ER that night.    Root cause analysis showed patient had known adrenal insufficiency and had stopped taking her meds because she disliked the side effects. She never told the ER staff or physicians about her diagnosis.  Pt had 2 episodes of hypotension prior to coding.   If you consider adrenal insufficiency you have to give hydrocortisone.    Dan’s take home point: If you care about a specific pmh item you have to ask the patient more specifically about specific illnesses.  These days, many patients cannot give a good pmh.

Elise comment:  Recurrent hypoglycemia can be a clue to adrenal insufficiency.   Dan Miller comment: If a patient has hypoglycemia, consider adrenal insufficiency.  Ask about medications, check the lytes to see if pt has hypnonatremia/hyperkalemia which is a marker for adrenal insufficiency.

Harwood comment:  In setting of hypoglycemia think adrenal insufficiency, oral hypoglycemic overdose, insulinoma, inborn error of metabolism, insulin abuse. ETOH in kids can also cause hypoglycemia.

Case 3

60yo male on ASA for CAD.  Recent ankle surgery.  Presented to ER after syncopal fall and head injury.  Dan got the sign out.   After sign out the departing team started heparin for syncope without telling Dan.  Pt ended up having a small subdural that was missed on initial CT.  Subdural increased and pt had to go to OR in AM.   There was a discussion of the dangers of sign out to patient safety.

Josh Eastvold   Class of 2010  EKG Ischemic Patterns

ST depression is a clue to ischemic EKG’s,  both STEMI equivalents  and subendocardial ischemia.  Localized ST depression is strongly indicative of STEMI or STEMI equivilant.

Subendocardial ischemia does not localize in the ECG.   Subendocardial ischemia is diffuse usually in infero-lateral regions.  Patients will have diffuse st depression.

 ST elevation in AVL may be very subtle.  So compare it to the TP segment.

Localized inferior st depression, look for st elevation in high lateral leads.   Also look for inverted t wave in V6.

Posterolateral STEMI  Look for inferior st depression, st elevation in 1,avl and st depression in left precordial leads (V1-3).

True posterior STEMI has horizontal st depression flowing into an upright large T wave (mostly in left precordial leads V1-3).  Pt also has tall R waves.    If st depression is more prominent in right percordial leads (V4-6) it is more likely to be subendocardial ischemia.

High risk pattern:  Twave inversion in AVL and ST depression in 1 and AVL.

If unsure if EKG is ischemic: 1. get multiple serial EKG’s.    2. Get bedside echo.   3. Consult cardiology.

You never see localized ST depression in a non-STEMi or STEMI equivilant.    Benign early repolarization never localizes to the inferior leads.

Inferior STEMI’s are frequently seen in association with posterior STEMI’s

Only 50% of anterior STEMIs have reciprocal changes.

New tall t wave in V1 is a marker for  ischemia.

There was a discussion lead by Erik Kulstad that there is not strong data to support doing caths on patients who have NSTEMI’s.   Josh Eastvold said his reading of the literature supports doing caths on true posterior MI’s.

Early repol in the anterior leads is always associated with normal to prominent r waves.  If r waves are poor anteriorly it probably isn’t early repol.

Many of the difficult EKG’s Josh showed were made clearer by getting serial ekg’s.  A big point of the lecture was get serial ekg’s.

QTC less than 390 is rarely a STEMI.  (Nice pearl)

If you see a dysmorphic QRS  “small ditzel QRS” it likely identifies an ischemic lead.

Tachycardia, with T wave inversion in leads 3 and V1-3 without T wave inversion in 1 and AVL  think PE.

 PE EKG

You can increase the EKG gain to highlight the inferior and high lateral leads that have subtle changes.

Jason Tomasello  Class of 2010   Atrial Fibrillation

Treating afib with rvr: First look for treatable causes.  Start treatment with Cardizem.   Magnesium may also work well.  Mag works particularly  well in holiday heart.

Treating afib in the setting of sepsis:  Digoxin was thought by the attending in the audience to be the best option.

Animated discussion among audience about safety of digoxin:  Most attendings felt digoxin was safe when given to sick elderly patients cautiously for rate control. 

Treating afib in the setting of CHF: Diltiazem has some data to back it’s safety in this scenario.  It is fairly neutral on blood pressure.

 Afib with RVR and hypotension:  Jason presented data that Amio bolus and drip may be preferred in this setting.  Push dose pressors may raise BP enough to give meds.   Other option is cardioversion.

Josh Eastvold suggested mast trousers in this scenario.  This suggestion was met with healthy laughter. 

Harwood comment:  Afib is notoriously hard to cardiovert.  You need to use high electrical doses.  You can use the defib paddle to apply extra pressure to the zoll pad to improve electrical contact when cardioverting afib.   Erik Kulstad: You can also press with your hand as there is no risk of being shocked.

Josh Eastvold comment:  Hilarious but important anecdote.   In the heat of battle, V-Fib can make you stop thinking and not be able to hook up the defibrillator zoll pads.   Using the paddles and not using the zoll pads may be simpler and faster when a patient suddenly goes into V-fib.

Afib with RVR and hypotension failing cardioversion:  High dose amiodarone (150 bolus followed by second 150 bolus and then a 2mg/min drip) is a consideration.  Get Cardiology on consult if you are considering doing this.

Dan Miller comment:  You could load with amio or procainamide and then retry cardioversion of unstable afib.

Target Heart Rate for rate control of afib with RVR is 120 or less.  You don’t have to get to a heart rate of 80.

Lovell    Patient Safety  

Health Care Error: preventable adverse event or effect of care whether or not it is harmful.

Near miss: Error that did not reach the patient

Causes of Error include human factors, biases, medical complexity, and system failures.

The organizations that drive patient safety:  #1 the Government.   They  are  creating positive and negative financial incentives to drive safety initiatives.  The ACGME demands that residents are involved in patient safety programs.   ACMC has committed to a clinical environment with zero preventable errors by 2020.

Dan Miller comment: There are a lot of unintended consequences that can result from the drive to be error free.  Elise reply: Agree.  Examples such as over use of antibiotics for pneumonia or avoiding complicated or sick patients to avoid any risk of complications/errors.

Girzadas comment: At CORD meeting the Director of the ACGME presented data that the error rate that resident experiences in his training program (the level of patient safety in the training program) stays with him his entire career.   Physicians continue to practice their entire careers at the level of patient safety they learned in their residency training. We thus have a huge responsibility to make our training program as safe for patients as possible. Not just for our current patients but for all our residents future patients.

 

Conference Notes 4-23-2013

Conference Notes   4-23-2013

Please consider donating to our EM Foundation to benefit resident education/development.  Thanks for your consideration/generosity.
/em-foundation/

Patel       M & M

Pt with pneumonia, sepsis and hypotension.    Central line placed for management of hypotension.

Central line was placed in carotid artery.  This was not identified until pt went to ICU.   Initial CXR reading did not identify misplacement.   CXR image was not visualized by ED physicians, only the report was reviewed. 

CVP complications  range from 5-19%.    Femoral lines tend to have a higher rate of mechanical complications (arterial puncture, malposition of catheter) than IJ and SC lines.    IJ and SC lines have similar complication rates (arterial puncture, malposition of catheter, pneumothorax).   Infectious complications are thought to be higher in femoral lines but there is some controversy about this.

5 steps to  Reduce infections of central lines: hand hygiene, chlorhexidine prep, barrier precautions/sterile gloves,gown,drape,mask, avoid femoral approach, and remove  lines as soon as no longer needed.

Identifying venous vs arterial line placement: color of blood and pulsatile flow are non-sensitive and non-specific especially when patients are septic/hypoxic/hypotensive.   Pressure transducer is a much more accurate way of identifying arterial flow but it is harder to do.    An easier trick is to hook up an iv line flushed with saline and connected to a liter bag to the catheter and hold it vertically over the chest.  If the column of blood reaches 10cm above the patient's chest, it is likely arterial.   Another option is to do an ABG of the blood from the line.  Elise comment: ABG is a fast and accurate way to identify arterial placement of a central line.

U/S guidance has significantly lowered the incidence of arterial sticks.  However arterial puncture may be missed on a short axis view.    A long axis view may improve your ability to identify arterial puncture or the needle passing thru the back wall of the IJ.  (Blaivas study 2009)

The operator needs to personally visualize the radiology images after all procedures. 

Elise comment:  If unsure of location of line, replace the wire and shoot a quick cxr to confirm location prior to dilating the passage.    Barounis comments: U/S can help you see where the wire and line is located.  If you inject 10ml of saline through the line with an U/S probe on the heart you should see a “splash” in the right atrium on U/S.  Recent research has suggested that Infectious complications and DVT are not more common in femoral lines.  Harwood comment: The most common problem is that the needle passes through the IJ into the carotid.  He has also seen an uncommon location for a central line, the azygos vein.

 

 

Central line in the azygos vein.

Fort      Trauma Lecture

23yo female transferred to ACMC following a T-strike MVC.  Pt was hypotensive and had a pelvic fracture that is bleeding out through a wound in the vagina.     Harwood comment:  If you are working in a non-level 1 hospital and a patient has a pelvic fracture always transfer that patient.  All trauma centers will accept pelvic fractures.

  Massive transfusion protocol initiated and pelvic binding applied.

Salzman comments: Patient is unstable.  Vaginal bleeding is a big problem.  FAST scan is critical in this situation.   There is a high incidence of bladder and urethral injuries with pelvic fractures.  Steve felt that you can make a case for gentle placement of a foley in a male patient with blood at the meatus prior to doing a urethrogram.   If you hit any resistance stop attempt.  Girzadas comment: I would do a urethrogram prior to placing a foley in a patient with suspected pelvic fracture and blood at the meatus.    Female patients are a different story.  There is less risk of urethral injury due to shorter urethra.  

Salzman comments: Vaginal laceration makes it harder to control pelvic fracture bleeding.   Binding the pelvis decreases the pelvic volume (potential space for bleeding) but with a laceration you can’t easily decrease the potential space the patient can bleed into.    Pelvic binding needs to encircle the iliac crests.     For IV access in a trauma resuscitation, triple lumens are bad, they do not provide large volume infusion capacity.  Nothing beats a 14g angio in the antecubital fossa (short and large guage catheters have the highest flow rates).   Cordis is next best.  IO may also have value but flow rates are alittle slower than large bore iv's or cordis.   FAST scan is important to identify significant intra-abdominal bleeding as a cause of shock vs. shock from pelvic fracture.   Key Decision is does patient go to OR first or IR first?  She will eventually need to go to both.  No role for CT scanning in the unstable pt with a pelvic fracture.  If FAST shows intraperitoneal blood, pt will go to OR first and IR second.

Harwood comment:  Pt should be intubated prior to going to IR or OR or if being transferred. 

Xray shows bad ass pelvic fracture with diastasis of the pubic symphasis and left SI joint widening. FAST showed no intraperitoneal blood.

Resident comments: Give TXA in this situation.   Salzman agreed and said patient needs to go to IR.

Salzman comment: If you decided to do a DPL, use a supraumbilical and open approach.   If you initially aspirate blood, you know the injury is intraperitoneal and you are going to the OR.  IF no blood on aspiration, you are going to IR for the pelvic fracture.

IR embolization of pelvic  arteries was performed.

Cystogram after patient is stable.  Instill 300ml of contrast under gravity into the bladder prior to getting images.

Pt had ORIF of pelvis.   Pt had repair of vaginal wound. 

Morel-Lavellee Lesion:  proximal thigh ecchymoses associated with severe pelvic or lower extremity trauma increases risk of infection of pelvic fracture.

They typically occur as a result of the skin and subcutaneous fatty tissue abruptly separating from the underlying fascia.The initial injury represents a shearing of subcutaneous tissues away from underlying fascia. The initial potential space created superficial to the fascia is filled by fluid of variable make up ranging from serous fluid to frank blood. The collection may then spontaneously resolve, or become encapsulated and persistent. It classically occurs over the greater trochanter of the femur 1. Although strictly speaking a Morel Lavallée lesion is only over the greater trochanter, similar biomechanical forces to the lumbar region, over the scapula or the knee can result in identical lesions 1,3.  (Radiopedia.org)

 

   Morel -Lavellee Lesion R hip  on CT

 

Morel-Lavellee lesion left hip Clinically

Bottom line:  Pelvic fracture with hemodynamic instability: bind pelvis/do FAST/resuscitate with fluids and blood/plasma/platelets.  If FAST is positive go to OR.  IF fast is negative go to IR.       Pelvic fracture that is hemodynamically stable: Go to CT and may need IR.        Harwood/Elise comment: If you are in a non-Trauma Center ED, transfer the unstable pt with a pelvic fracture out of your ED ASAP.    Start PRBC’s/FFP/Platelets ASAP for the hemodynamically unstable pt with a pelvic fracture.  Intubate early as well.

Collander/Williamson   Oral Boards

Case 1   TCA overdose         Critical Actions: Assess airway, IV fluid, identify overdose and substance, treat with IV bicarb, admit to ICU.  EKG showed wide terminal R wave in AVR and QRS widening diffusely which are classic findings for TCA overdose.   Pt also had signs of anticholinergic toxidrome.

 

EKG of TCA Overdose.  Note R wave is wide and tall in AVR.  Also QRS is widened in all leads

 

Case2    AAA        Critical Actions: Recognize shock,  use U/S to identify AAA,  start resuscitation with IV fluids and PRBC’s, consult vascular service for emergent surgery.  AAA more common in older men who are smokers.

Case 3   Chemical Exposure(dishwasher soap) to Eye of child        Critical Actions: Irrigate the eye, check visual acuity after irrigation, check ph after irrigation, consult opthalmology.   Andrea Carlson comment: If a child orally ingest dishwasher or clothing detergent, it can cause severe respiratory injury or gi injury.  These are strong alkali’s.

McKean    CPC Presentation

Williamson    CPC Presentation

Kettaneh    Ketamine for Continuous Sedation  ICEP Research Presentation

Dissociative agent that provides sedation and analgesia. Preserves respiratory reflexes/breathing.

Side effects include: emergence phenomenon, salivation and bronchorea,  used to be thought to increase ICP but this has been called into question.

Retrospective study at ACMC. Enrolled patients admitted from the ED to the ICU. This was an overall sick/hypotnesive group of patients. Ketamine was used in the ED and ICU for continuous sedation.

Don’t want to give away all the data prior to presentation but bottom line:  Average dose of 2mg/kg/hour.    It worked well.  Patients were generally well sedated. There were 2 cases of agitation requiring a change to benzos. There were 2 cases of afib that may or may not have been due to ketamine.   Amazingly this is the first study to assess ketamine as an agent for continuous ICU sedation. 

 

 

 

Conference Notes 4/16/13

Kettaneh/Felder    Oral Boards

Case 1.  Syncope due to Brugada syndrome.  Pt also fractured humerus due to fall.  Critical actions: Identify brugada and consult cardiology. Splint and manage fx.  Brugada syndrome arrhythmias usually occur at rest.

Three types of Brugada waveform. Three distinct types of ST segment elevation have been described. In type 1, the ST segment gradually descends to an inverted T wave. In type 2, the T wave is positive or biphasic, and the terminal portion of the ST segment is elevated ≥1 mm. In type 3, the T wave is positive, and the terminal portion of the ST segment is elevated <1 mm. Arrows denote the J-waves.

Case 2.   Seizing patient due to hyponatremia from post-partum pituitary necrosis.  Pt had intra-partum hemorrhage with hypotension and transfusion. This resulted in pituitary infarct/necrosis.   Pt has been having difficulty lactating and has been weak.  Critical actions: Hypertonic saline, antibiotics for uti, hydrocortisone.  Consult appropriately.   Sheehan syndrome consists of pituitary necrosis due to ischemia in the peripartum period.  

Case 3.   Seizing NH patient due to hypoglycemia.  Pt accidentally received  a dose of insulin intended for another patient. Critical actions:  Identify hypoglycemia, IV glucose, feed, check serial blood sugars. 

Elise comment: If there is a specific fx that causes a known neuro-vascular injury then specifically check for this injury.  Example: midshaft humerus fx is associated with wrist drop.  Specifically tell the examiner you want to examine for wrist extensor strength.

Harwood comment:  You can get an insulin level on a patient to see if they received too much insulin.  There can be varying severity of Sheehan’s syndrome.  Mild cases can be difficult to diagnose.   There is a website brugada.org that can give you some guidance on how to manage these patients.   Incidental brugada cases identified in the ED should be referred to an EP specialist to see if an arrythmia can be induced.   IF so, they will need an AICD.

Girzadas comment: Be alert for hyponatremia and mild hyperkalemia in a weak or hypotensive patient. This is usually adrenal insufficiency. Get a serum cortisol level and give hydrocortisone in stress doses. 

 

Chandra    M&M

Pt with fournier’s gangrene.   Rapidly progressing necrotizing fasciitis of perineal/perianal/genital region.  Usually in men around age 50-60.   Diabetes and obesity increases the risk.  Indwelling catheters, etohism, immunosuppression also increase risk.

Can be caused by local infections like abscess, fissure, diverticulitis, etc.   10% of cases are idiopathic.

Look for scrotal pain, swelling, fever. Patient’s may have severe genital pain.   Check for subcutaneous crepitation.

94% of cases involve the srotum,  47% involve the penis, and 35% of cases involve the perineum.

Fournier’s severity index: Basically SIRS plus lab abnormalities correlates with increased mortality.   Elise comment: This index looks like unvalidated BS.

U/S can show subQ air but is less sensitive than CT.  CT is most sensitive for subQ emphysema and can show source of infection and extent of spread.   Harwood comment: Get fine cuts through scrotum and perineum.  Less concerned about the rest of abdomen.

Treat with surgical debridement.   Give triple antibiotics.  Hyperbaric o2 is controversial and is used after surgical debridement.   24 hour delay in surgery increases mortality by 11%.  Mean hospital stay is 30 days.

Pitfalls: Have high suspicion for this rapidly progressive diagnosis.  Sign outs increase risk.   Push back against consultant bias that downplays disease process.

Girzadas comment: During sign out make sure there is some back and forth discussion between teams about these high risk patients.   Harwood comment: Be careful to not just say “nothing to do”.  Rather you need to say, you have to re-examine the patient’s scrotum.   Question whether the antibiotics have been written for repeated doses if they are boarding for a prolonged time in the ED.  Elise comment: Identify your sickest patient and highlight the need for further attention.    DenOuden  comment:  Bedside sign out may be helpful in these high risk cases.   Harwood comment:  These guys need serial exams because it can be “awe inspiring to see how fast this can progress”.  Christine comment: Ct is very important in these diagnoses.   Harwood comment: MRI is the most sensitive test for necrotizing fasciitis.   

Paarul comment: Prior to sign out review your orders for critically ill patients.

Harwood comment:  This case was due to a system error.   Our EMR takes so many clicks to place orders, it increases your risk of error.  Elise comment: It is ok to push back professionally against consultants if you feel strongly the patient needs a certain intervention.   Kelly Williamson comment: We train for worst case scenarios and are constantly alert for them.  Consultants frequently down play a disease process.  You have to be aware of this difference in perspective.  Willison comment: Inturruptions also increase our risk for error.  Joan Coghlan comment: signouts are made more difficult also by the arduousness of the EMR system.

 

Kettaneh    Tranexamic Acid in the ED

TXA is a synthetic analog of lysine.  It competitively inhibits the activation of plasminogen to plasmin.

Common minor Adverse effects: abdominal pain, arthralgia, headache, fatigue.

CRASH-2  study:  40 countries with 20,000 trauma patients.  Showed lower all cause mortality.

MATTERs  study: showed lower mortality in TXA group.

  ICH study: showed trends to less ICH growth and lower mortality.

Re-analysis of CRASH-2  showed the earlier you give TXA the better the effect.  Want to give in less than 3 hours.

Give to Trauma patients with hypotension or tachycardia  or expected  need  for transfusion.   Until further research, don’t give to isolated head trauma patients.

Dose is 1g IVPB over 10min followed by 1g IVPB over 8 hours.

PharmD comment:  This is a benign drug.  It is not a pro-coagulant.  It only stabilizes the clotting system.   Elise comment: This drug provides a modest but real benefit in bleeding trauma patients   There was consensus  that we should be using this in multisystem trauma patients with bleeding, hypotension, tachycardia, or need for transfusion. 

 

Kettaneh          Case F/U

63yo male with presyncope.  Sob and diaphoretic after lifting a pallet.    His wife made him go to the ER.

Which is a clear marker for bad pathology.  Harwood comment: Another marker for bad pathology is when long time smokers spontaneously stop smoking.  When a patient tells him they recently stopped smoking on their own, he gets worried.    Vitals showed tachycardia.  EKG showed sinus tachycardia with subtle inferior st changes.

Pt became hypotensive in the ED.  Bedside echo shows moderate pericardial effusion with good contractility.  No chest or back pain.   Dimer is elevated.   Second EKG looks better.

To diagnose cardiac tamponade , tachycardia is almost always present.  JVD is commonly present.  Electrical alternans is an EKG finding specific  but insensitive for tamponade.

 

Electrical alternans.

U/S will show diastolic collapse of RV when tamponade is present.

CTA of this patient showed an aortic dissection and no PE.

Type A in Standford classification involves ascending aorta.   Type B means only descending aorta.

This patient had a painless Type A dissection.  6% of aortic dissection patients have no pain. These patients with painless dissections are more likely to have syncope, stroke symptoms or CHF symptoms.

Patient went to OR and had repair of ascending dissection and aortic valve replacement.

Nick felt the dimer may have saved the patient’s life.   Kari Tekwani recently diagnosed a relatively painless dissection on another patient and she also felt the dimer helped pick up the dissection.

Harwood comment:  If you aren’t sure if the diagnosis is pe or dissection go with the CT PE rather than CTA study.   You probably won’t miss a significant dissection on a CT PE study.   If you do a CTA aorta study, you will miss many pe’s.   

Watts     Vent Management Software Demonstration

Cash    5 Slide F/U

91 yo female with altered mental status.   Dexi was 56 in the field.   Pt was hypothermic and hypertensive.  Cachectic and dehydrated.

Labs c/w urosepsis.   Pt became more hypothermic and became hypotensive.   Diagnosis was urosepsis.

Started on Cetriaxone and Levophed.     IM brought up possibility of myxedema coma.   EM clinicians considered diagnosis but felt the expected doughy appearance of a myxedema patient  was not present.   Pt’s TSH was later found to be 250.  Diagnosis was made of myxedema coma. 

  Hallmarks of myxedema coma are: mental status change, hypothermia, bradycardia, hyponatremia, hypoglycemia, hypotension, and precipitating illness.

Treat with levothyroxine, hydrocortisone to cover possible adrenal insufficiency. Manage hypoglycemia, manage hypotension, provide passive re-warming, and monitor for arrhythmias.

Beckemeyer    5 slide F/U

79 yo female with sudden onset headache and neck pain with emesis.   Pt has afib and is on Coumadin.  

ED clinicians ordered CTA of head and neck.  Pt was found to have a right MCA aneurysm with diffuse SAH.   Pt was given FEIBA to reverse Coumadin  and then went to neurointerventional lab and had aneurysm coiled.  Pt was dc’d home with no focal deficits.  Badabing Badaboom!

 

 

Conference Notes 4-9-2013

Conference Notes 4-9-2013

Please consider donating to our EM Foundation to benefit resident education/development.  Thanks for your consideration/generosity.
/em-foundation/

 

McKean        Trauma Lecture

69 yo patient with htn/dm in a significant mvc.  Pt is transiently hypotensive on arrival but still talking. There is a BP difference between upper extremities

Harwod comment: Need an EKG in this situation in addition to usual trauma resuscitation.  BP difference in arms could be due to aortic injury or chronic vascular disease.

CXR shows wide mediastinum.  Lungs look clear.  FAST exam is negative.   Thought at this time is probable aortic injury.

Salzman comment: Pt is using his abdominal muscles to breathe and pt is going to CT scan.  Probably need to consider intubation.  Salzman then made an argument against tubing at this point due to risk of worsening hypotension with induction/worsening vascular return and taking more time prior to sending to CT and burning the time that patient has some relative stability.   Difficult call on what to do.  There was general agreement that Trauma team made the right move and sent patient to CT without intubating prior.  

Harwood comment:  Need to evaluate successive data points over time to decide what to do.  If all clinical data is trending down then you have to tube.  If the data points are trending flat and not particularly downward then make a run to the scanner without tubing prior.

Salzman comment:  Indictions for evaluation of aorta are decel injury of 35 mph or greater, T strike MVC of significant force, fall of varying height depending on patient’s age/body habitus/specifics of fall.  Most aortic injuries occur just distal to the left subclavian artery.  There is a ligamentous tether near this point.

CT showed aortic injury, liver and splenic injury.  Pt also had a pneumo.  (Harwood got Kudos from Salzman for picking up the subtle pneumo  from the back row on CXR that was shown during this presentation.)  CT showed intraperitoneal blood.  So there was a discussion of why the initial FAST was negative.   Consensus was to do serial FAST exams because the test may be dynamic.   Barounis comment:  Trandelenburg position may bring intraperitoneal blood into Morrison’s pouch and make it more visible.

Pt coded.   Salzman comment: Bilat chest tubes placed. Pt had right sided pneumo.  Left side was tube was placed to see if aorta ruptured into left hemithorax.  No blood in left thorax.   Pt was taken to OR.   Belly was full of blood, far in excess of what the CT and the FAST demonstrated.  Pt coded again in OR and died.

In the end, This patient was not salvageable with his comorbidities and his severity of injuries.  Everything was done rapidly and appropriately.

Take home points:  Early transfusion, use cell saver and level 1 transfuser.   Transient hypotension is a marker of instability.  Be sure to save any blood coming out of chest tubes to transfuse back into patient. Do repeated FAST scans in unstable patients. An early FAST may miss smaller amounts of intraperitoneal bleeding.

Harwood comment: Bleeding may have slowed when pt was hypotensive resulting in negative FAST.  When pt was resuscitated his bleeding increased resulting in CT findings of intraperitoneal blood and later more severe OR findings of massive intraperitoneal bleeding.  

Salzman comment: For ER docs working in rural environments the decision to transfer this patient should be early.   He would start the transfer process as soon as he saw the initial CXR showing the wide mediastinum.

Gottesman               Bleeding Disorders

Hemophilia A: Deficiency of factor 8.

Hemophilia B: AKA Christmas disease is a deficiency of factor 9.

Mild disease 6-40% of either factor.

Moderate disease 1-5% of either factor

Severe disease <1% of either factor.  These patients may spontaneously bleed.

Hemarthrosis/muscle bleeds:  Warmth, paresthesias or pain.  Re-assess frequently for developing compartment syndrome.   Treat pain with opioids.  Avoid NSAID’s and ASA.

For oral mucosal bleeding you can replace the particular factor and give a mouthwash using Tranexamic acid.  Pharmacy can mix up tranexamic acid in an oral solution for an 8.4% concentration.

Head bleeds: Give Factor replacement prior to getting CT.

Iliopsoas bleeding can cause vague back or abdominal pain/paresthesias.   Elise comment: is there something specific about this muscle group?  Allisa: It can hold a lot of blood and is not directly visible/palpable so it has to be evaluated by CT. So you need to be aware of this.

There was some discussion of whether a hematologist needs to be consulted to give factor replacement.   All faculty felt this can be given unilaterally by an ER doc.

Avoid IM injections in hemophiliacs.  It can cause bleeding.

Hemophilia treatment centers have been shown to reduce mortality by 40% over six years.  Erik comment: Hemophiliacs know this fact and tend to go to those centers for their treatment. That’s why we don’t see that many of these patients.

Barounis comment: Simple way to remember treatment is 50u/kg factor 8 for serious bleeds (Head, GI, other life threats) and 25u/kg factor 8 for less serious bleeds (hemarthrosis, hematoma in soft tissue, etc).  For factor 9 replacement double these doses to 100u/kg and 50 u/kg.   Elise comment: Remember that epistaxis is considered a serious bleed.   Allisa: giving factor replacement too fast(faster than 2 minutes)is  painful for patient

FFP can be used if you don’t have factor 8 or 9 to give, or you don’t know the specific cause of the coagulopathy. However, volume precludes using this FFP effectively to correct factor deficiency.  Elise comment: Head bleed with undiagnosed clotting disorder give FEIBA.  Allisa agreed. Volume is not an issue with FEIBA. .  Pharmacist: FEIBA infusion rate is 2u/kg/min.

Patients who have inhibitors treat with FEIBA.

Mild bleeds can go home.  Admit for life threatening bleeds, concern for compartment syndrome, pain control, needing 3 or more doses of factor replacement.

There are people who develop acquired hemophilia due to autoimmune cause. Most commonly in post-partum women over age 60.

Von Willebrand disease: most common bleeding disorder.  Mucosal bleeding/bruising.   Minor bleeds treat with 50u/kg of vW/F8 complex.    Major Bleeds 100u/kg .

In kids with no history of clotting disorder who have large hematomas following  mild injuries ask if there is any family hx of clotting disorders.

You don’t need to have documentation that someone has hemophilia to treat them.  If they say they have it, believe them and treat them as indicated.  If they have their own factor replacement with them use it. 

Ptt should be abnormal in most hemophilia patients with severe disease.

Erik: Bleeding time is no longer available at ACMC.  Substitute is platelet function assay.

 

Clotting pathways from Up to date

Herrmann/Paquette    Oral Boards

Case 1.  50yo female with Pneumonia/afib/thyroid  storm   Critical actions:IV abx, beta blocker, PTU, Potassium iodide, and glucocorticoids.   Lovenox for afib.

Case 2.   4wo child with meningitis    Critical actions: Abx choice is amp and cefotaxime.  Cefotaxime instead of ceftriaxone at this age to avoid displacement of bilirubin from protein binding sites.   There was Discussion of whether vanco should be given in this case.  Consensus that it was not needed unless you have some indication of MRSA.

Discussion between Elise and Harwood and Barounis about whether a well appearing 6 week old infant who has a clear cut ua showing infection also needs LP.   Elise referenced study showing that @1% of infants under 28 days had concomitant UTI and meningitis.  These infants with both illnesses  were ill appearing.   No children over 28 days who were well appearing had concomitant meningitis and UTI.

Case 3.   21 you male with appendicitis     Critical actions: Pain control, IVF and have surgery take patient to OR

Schroeder    Parental Questions

>50% of parents believe a fever is present even if the child’s temp is less than 38C.

60% of pediatricians believed that a fever >104 was harmful.

Parents think a fever is a disease not a symptom.

Fever and heat stroke a entirely separate entities.  

Currently it is thought that a fever that is particularly high( >104) is not associated with higher risk of bactermia.   Current vaccines have decreased the incidence of pneumococcus. Pneumococcus previously was a cause of higher fevers and higher wbc counts and thus kids with high fever before pneumovax was introduced were more likely to be bacteremic. This is no longer the case.

Tylenol vs motrin: Relatively equal efficacy.   Head to head studies have had variable results.  Alternating antipyretics every 3 hours carries some risk of dosing misadventures by parents.

Girzadas comment: Easy dosing short cut is 1 tsp of either acetaminophen or ibuprofen for every 10 kg.  You can fine tune beyond that with the conversion of  1ml for every 2 kg.  Example 15 kg kid gets 1.5 tsp.   17kg kid gets  8.5ml.

Simple Febrile seizures are harmless.  They are due to circulating cytokines.  No need to do any further work up or LP if the child looks well after the seizure.  Do your usual eval for source of seizure.  Dr. Schroeder said he treats a simple febrile seizure as if it didn’t happen when he is evaluating the child.  He does however validate the parents’ concerns about their child.  He understands how scary it can be for a parent to see this happen in their child.

Vomiting kids:  Tell parents to have child drink with spoon for the first hour.  If they don’t throw up they can move on to drinking with a cup.   BRAT diet is no longer recommended.  Restart regular diet as soon as possible.   Ice cream is a good choice.  Avoid spicy, fatty, sugary foods.  Sugar free foods can worsen diarrhea.  Routine restriction of milk is not recommended.   Probiotic can be helpful for diarrhea. Florastorkids 250mg po q day.  Yogurt bacteria may not survive into the child’s gut.

Constipation:  Unhurried toilet time scheduled every day.  Take advantage of gastrocolic reflex.  Have child sit on toilet 5-15 minutes on toilet 20-30 min after breakfast and dinner.  Miralax is Bill’s go to medicine.  More palatable to kids.  Give until child is having bowel movements then wean them off over a week by decreasing the dose by half every 3 days. Harwood comment: Miralax is a very safe drug. He gave the example of a child he saw on miralax for a month who was having diarrhea and child had no electrolyte abnormality. 

Feeding:Overfeeding is very common in young children and is frequently the cause of GERD. Don’t give water to childen under age 2months. They can develop hyponatremia very easily. Have to be sure parents are not over-diluting their kid’s formula.

Teething: Give the child a wet washcloth that was cooled in the freezer to gnaw on.   Topical teething gels can cause methemoglobinemia from the local anesthetic.

Crying: 6 weeks is the height of newborn fussiness.  The average child cries for about 3 hours a day at 6 weeks.   5S’s for comforting the child: swaddle, lie on side, soothing sound, swing (motion), sucking.  You can use in step wise cumulative order.   Bill’s key was a good swaddle.   Gas drops have not been shown to help colic. Infant homicide increase at the 2nd week of life and peaks at 8 weeks of life which correlates with the natural history of colic.     

Strep throat is uncommon under age 2. False positive rate equals or exceeds the true positive rate.  Risk of rheumatic fever due to strep is very low under age 2.  You don’t have to treat under age 2.

Umbilical hernias: 90% close by age 4.  Complications are rare.  No surgery under age 5-6. If greater than 1.5 cm then unlikely to close.

Frazier        5 Slide F/U

21 you female with chest pain after tonsillectomy.  CXR showed pneumomedisatinum.   Barium swallow suggested by Thoracic Surgery showed hypopharynx perforation.  CT showed mediastinal air as well. Pt started on Unasyn. Repeat esophogram 3 days later showed resolution of perforation.  Pt never spiked a fever.  She did well.   Review of case reports demonstrates that It is not known if this complication is usually due to intubation or surgery.  Treatment is observation for hypopharyngeal perforation.  If pneumomediastinum is from esophageal perforation, pt requires surgery.   This differs from spontaneous pneumomediastinum which can be discharged without treatment.

Harwood comment: this is most likely due to surgery not intubation.  This has also been reported in the dentistry literature.

Balogun  5 Slide F/U

9 mo child with rash for 2 days.  Rash around mouth and distal extremities.  Pt had been brought to 2 separate outside ED’s before this ED visit and was on augmentin and mupirocin and bactrim.   Positive Niklolski’s but no mucosal lesions.   No mucosal lesions differentiates Staph Scalded skin syndrome  from Steven Johnson’s syndrome. SJS has mucosal lesions 98% of the time.  This patient had SSSS.  SSSS has perioral crusting and kids are well appearing.  SSSS is due to exfoliative toxin.  Treat SSS with IV  fluids using parkland formula and give IV clindamycin. 

 

 

Conference Notes 4-2-2013

Conference Notes 4-2-2013

Please consider donating to our EM Foundation to benefit resident education/development.  Thanks for your consideration/generosity.
/em-foundation/

EM-Peds Case Review

Protected Discussion but the following were take home points:

Don’t be squeamish about putting in an IO line.  Tell parents it is an IV in the bone.  If the kid is sick and no IV in 2 sticks then put in an IO.  Easy/safe/effective.

Also consider hydration through G-tube if one is present until you have IV access.

Give kids their maintenance meds while in the ER if their stay is prolonged. 

Contact PICU early and be careful to effectively convey the acuity of the situation.  The physician on the recieivng end of the  phone naturally tends to interpret the acuity as lower than it actually is.  You can always have the ED attending contact the PICU attending to ask for evaluation/help managing  the patient in the ED.

Parents of chronically ill kids may downplay the severity of the child’s illness. They are so used to recurrent problems they may underestimate the level of their child's acuity.  Be cautious of this phenomenon and  stick with your instincts and carefully consider the patient’s vital signs. 

Be very attentive to a patient with a heart rate over 180 or with grunting respirations.   These are two of Dr. Roy’s red flags.  

In ill appearing febrile kids, give empiric antibiotics early and often.

Give stress dose IV steroids  for ill kids with risk for adrenal insufficiency. 

 Fort/Herron                    Oral Boards

Case 1:  Near Drowning/Hypothermia   Critical actions: External rewarming, CXR to eval for aspiration, Observe for developing pneumonia.   Initial CXR in these situations can be normal and evolve. 

Case 2:  Pt with non-specific complaints who is found to be a victim of Domestic Violence    Critical actions:  Wade through patient's nonspecific complaints and Ask patient about possible domestic violence, check UCG, recognize depression, give tetanus shot, consult social services.    Domestic violence can affect all socio-economic groups.   Common presenting times are nights and when male partner is distracted by things like super bowl and march madness.   Need to offer social worker services. Pt can refuse.  In Illinois, You don’t need to report to police if pt does not want her case reported.    Elise comments: Very controversial issue.  Reporting can cause harm to patient.  In California EP’s are mandated reporters  and this can be problematic. It can potentially leave patient in a very vulnerable situation.  In Illinois, don’t report If patient does not want you to.  We are not mandated reporters in Illinois. For Oral boards, 2 rules of thumb: all kids have been abused and all women are pregnant or abused.

Case 3:  Ecclampsia resulting in low speed MVC    Critical actions: Identify that the patient is pregnant, give Magnesium for seizures, Evaluate for traumatic injuries. 

Collander/ Barounis comment: Ecclamptic patients should all probably get CT scans of the head.

Harwood comment:  If a patient is shivering then they have mild hypothermia and they will be fine. If they aren’t shivering they have severe hypothermia and need more aggressive warming.   Neuromuscular blocking with paralytics will stop the muscular manifestations of a seizure but they will still be seizing in their brains.  You can’t just treat patient with paralytics.  Gotta  give them some anti-epileptic and probably also monitor with an EEG tracing to make sure seizure has been halted.  

Barounis/Collander comment: In a hypothermic patient, if tachycardia is present you have to consider some other pathologic process in addition to hypothermia ,like toxic ingestion.  Isolated hypothermia should make a patient bradycardic.  

Sayger /Katiyar/McGurk  Critical Care Billing and Coding

85% of EP pay comes from level 1-5 codes.

We generally are under-documenting critical care time.

Critical care is defined by medicare as patients that are hypotensive, Impairment of one or more vital system functions ,require prolonged bedside physician care, or are dying or at risk of dying.   Examples: afib with rvr, stroke, sepsis, ICU admits.  ICU admission is not required for critical care billing.  Examples of non-icu admitted  critical care would be severe asthma or CHF that has improved to the point of floor admission.  McGurk comment: If patient is discharged home it is unlikely they will bill for critical care.  Harwood and Katiyar comment: STEMI’s should be billed as critical care.   Mistry comment: You have to document clearly how you provided critical care.  You have to be do this right in case you get audited.  Your % of critical care patients should probably not exceed 5%.  Altman comment: Medicare can clawback your billings and potentially prosecute you criminally for potential jail time.   So you gotta make sure you are doing this right.   

Katiyar comment: we should be billing critical care better because critical care is the core of our specialty and what we feel is the most important.  The RVU’s for critical care are much more than the levels 1-5.

A lot of EP’s don’t bill critical care because we consider much critical care we do routine care.

Key documentation: critical illness/injury, high complexity decision making, total critical care time exclusive of billable procedures.   Sayger comment: you can’t just list results.  You have to document some analysis and interpretation of results and clinical care provided.

Mcgurk comment: Say in your note, “I spent X minutes providing critical care for this patient”

Discussing care decisions with family members of an unresponsive patient can be included in critical care time.  This discussion needs to be documented.  Simple updates of patient’s status do not count for critical care.  Harwood comment: discussion of a LET form would count as critical care.

Medicare Audits of charts look at severity of illness, documentation, high complexity decision making and time claimed.

 History limitation on top of first net charts, in effect, acts as EM caveat for the chart.

Permar    5 Slide Follow Up

12 day old male infant with HR=250.  EKG with narrow complex tachycardia.  Vagal maneuvers: ice, rectal temp had no effect.    Adenosine worked only transiently.    Echo at bedside showed structurally normal heart.  IV Amio drip started and pt converted within 30 minutes.  Pt had recurrent SVT in ICU and a procainamide drip was started in addition.  Next step would have been cardioversion but this wasn’t needed. Dose for cardioversion is 1-2J.  Dose for defibrillation is 2-4J.  Dx was persistent junctional reciprocating tachycardia.   This arrhythmia can be troublesome to convert.  Pt dc’d home on oral amiodarone.

SVT usually has a heart rate >220. 

Hemming  5 Slide Follow Up

34 yo male with back pain.  Urinary incontinence.  Pt had weakness and decreased sensation in bilat lower extremities.  

Main diagnosis considered was Cauda equina syndrome : urinary retention, saddle anesthesia (buttocks/perineum),  60-80% of patients have decreased sphincter tone.  Pt’s have variable motor  findings in lower extremites.

 

Pt had no canal stenosis on MRI.  His symptoms resolved spontaneously within 2 days. 

Signs of Non-organic back pain: inappropriate tenderness, pain with simulated axial loading, distraction signs, neuro findings not corresponding to neuro disease, over-reaction during physical exam.  

Girzadas comment: Be very cautious chalking findings up to psychiatric disease.  The EP’s job is to evaluate for organic cause of back pain.

Harwood comment: This patient should have gotten a psychiatric consult for this second visit. 

Herrmann comment: Check for urinary retention with ultrasound when considering cauda equina syndrome.

Altman comment: Use the Illinois narcotic prescription history online.   It is very helpful to decide how to manage the patient.

 

 

3/26/13

Conference Notes 3-26-2013

Please consider donating to our EM Foundation to benefit resident education/development.  Thanks for your consideration/generosity.
/em-foundation/

Herrmann    Trauma in the Pregnant Patient

Bedside U/S will give you a quick assessment of the fetus as far as fetal movement, heart rate and age.   Bedside U/S does not give you adequate info regarding abruption.   You need to do fetal monitoring to determine the frequency of contractions to guage the probability of abruption.  Normal heart rate and no contractions in 4 hours of observation has 100% negative predictive value for abruption.

Pertioneal signs are not as evident in the pregnant patient.   Blunt trauma has a higher mortality than penetrating trauma.  0.5 of fetal deaths result from “mild” trauma.

No single imaging study provides a radiation dose high enough to cause harm in a fetus.  So image as needed but no more than needed.

Kleihaur Betke test identifies fetal-maternal hemorrhage that exceeds the normal dose of Rhogam (300mcg).  If KB test is positive you need to give extra Rhogam.

Left lateral decubitus position when pregnant patient is supine off-loads the inferior vena cava and can treat hypotension.

Peri-mortem C-section: Best results when initiated within 4 minutes of maternal arrest.  Goal is to get kid out by 5 minutes of mom arresting.   C-section off-loads the IVC and may improve maternal survival,  CPR continues during C-section.   Make big midline incision (xyphoid to pubis).  Hysterotomy through upper uterine segment.   Cut through placenta if necessary.   Have two teams if possible to divide resuscitative efforts between mom and baby.

Harwood comments:  KB test is of less value in 2013 because most RH neg mom’s have gotten a rhogam shot as part of their standard prenatal OB care.   Each CT study has to be carefully considered in the pregnant patient.  Do  the studies that need to be done but no more than that.   If doing perimortem C-section make big xyphoid to pubis incision going around umbilicus. 

Girzadas comment: If you are looking for a bright line cutoff of which mom’s  need fetal monitoring after trauma my suggestion:  If a butterfly alights on a patient’s gravid abdomen and the fetus is >20weeks ega, they should have monitoring for 4 hours.    Dr. Omi also felt a very low threshold was indicated. 

Girzadas                    Study Guide   Arrythmias/Pharmacology

I gave the lecture so I couldn’t do detailed notes, but trust me the lecture was Awesome! :)

Antiarrythmic classification

  • 1. Sodium channel blockers
    • 1a   procainamide, disopyramide, quinidine, tricyclics
    • 1b  lidocaine, mexilitine, phenytoin
    • 1c   encanide, flecanide, propafenone
  • 2. Beta-blockers
  • 3. Potassium efflux blockers-  Amiodarone,  sotolol,  ibutilide
  • 4. Ca channel blockers
  • Unclassified-   Digoxin, Adenosine, Magnesium

There was some discussion on management of arrhythmias/QRS widening/hypotension due to TCA overdose

Harwood comment:  TCA caused QRS widening and hypotension can be treated by raising the ph alone. The Na in Sodium Bicarb is not necessarily the key factor.  Harwood then discussed the following abstract:

Abstract
Study objectives: We carried out this study to determine the effects of pH alteration on QRS width with administration of tromethamine, a non-sodium-containing buffering agent, in experimental amitriptyline overdose. Design: Prospective, nonblinded trial. Participants: Adult mongrel dogs. Interventions: Pentobarbital-anesthetized dogs were overdosed with amitriptyline 5 mg/kg followed by infusion at 1.0 mg/kg/minute until the QRS width doubled, then decreased to .5 mg/kg/minute until the end of the experiment. At two defined points of toxicity, the dose of tromethamine required to raise the pH to 7.50±.4 was given. pH and QRS width at a speed of 100 mm/second were measured
over a 30-minute period after each tromethamine dose. Data were analyzed with non-linear-regression analysis. Results: At toxicity 1 the mean pH was 7.32, with a QRS width of 11.6 mm. Two minutes after the tromethamine dose the pH rose to 7.51, with narrowing of the QRS width to 8.4 mm. At toxicity 2 the pH was 7.40, with QRS width of 10.6 mm. Two minutes after tromethamine, the pH rose to 7.49 and the QRS width decreased to 9.7 mm. Regression analysis showed a correlation
between pH and QRS width; as pH increased, QRS width decreased (P=.0001). Conclusion: Cardiac toxicity of amitriptyline overdose, as manifested by QRS widening, is reversible by pH changes alone.

Andrea comment: TCA’s are highly protein bound.  In overdose situations, changing the ph of the blood will increase protein binding and decrease toxicity.  The NA is sodium bicarb can act synergistically on the fast sodium channels. 

Mckean/Kerwin    Oral Boards

Case 1: Werniecke’s Encephalopathy.    Classic triad of Encephalopathy/ataxia/oculomotor dysfunction.is present only 1/3 of the time.  Critical Actions: Rule out toxic alcohols, consider disagnosis, give hi dose thiamine 500mg IV Tid. 

Case 2: Afib with WPW. Critical Actions: IV procainamide 17mg/kg no faster than 50mg/min.  Stop if pt gets hypotensive  or QRS widens by 50%.   Avoid calcium channel blockers or beta blockers or anything that would block the AV node.  That would increase conduction down bypass tract.  If patient is unstable with chest pain, hypotension, altered mental status or severe chf then cardiovert.

 

Hemodynamically stable monomorphic VT or pre-excited atrial fibrillation (ACLS, 2010): Loading dose: Infuse 20-50 mg/minute or 100 mg every 5 minutes until arrhythmia controlled, hypotension occurs, QRS complex widens by 50% of its original width, or total of 17 mg/kg is given. Follow with a continuous infusion of 1-4 mg/minute. Note: Not recommended for use in ongoing ventricular fibrillation (VF) or pulseless ventricular tachycardia (VT) due to prolonged administration time and uncertain efficacy.

Girzadas comment: consider using only procainamide on the boards for narrow/wide afib or wide complex tachycardia.

Harwood comment: give procainamide 1gram over 30 minutes for your initial managment.  This rate will work safely for the majority of patients. If they get better you are done. If they get worse you can shock them or give more procainamde if they have a higher BMI.

Case 3: Toxic Shock syndrome.  Fever/rash (sunburn)/desquamation/hypotension/3 or more organ systems.     Critical actions: management of shock/abx for staph/remove foreign bodies/surgery for abscesses or localized infections.      Kettaneh comment: pt had criteria also for TTP however  faculty felt platelets weren’t low enough for TTP.   Elise comment: Put the synapse in your brain that shock with a sunburn type rash is toxic shock syndrome.  Girzadas comment: think about TSS or Anaphylaxis in the hypotensive pt with vomiting/diarrhea who doesn’t improve with a liter of fluids.

 

Toxic shock rash from Up to Date

 

Toxic Shock Rash from Up to Date

 

Maletich/Gupta   Oral Boards

Case 1: Benign Intracranial Hypertension.  Critical actions: Do physical exam to find afferent papillary defect and papilledema, pain medication, head ct, LP to check opening pressure.    Treatment with acetazolamide is the medical management.   Nosek comment: It is unusual to see unilateral eye findings.  Rohit agreed.  He said findings are usually bilateral but there may be asymmetry of severity of findings from right to left eyes.

Case2:   Pyloric stenosis.     Critical actions: IV hydration, diagnose pyloric stenosis with U/S, consult surgery, admit patient.   Classic metabolic picture is hypochloremic metabolic alkalosis.  Hypokalemia takes a couple of weeks to develop. 

Case3:  CHF/Pneumonia, ARF with hyperkalemia.  Critical actions: Treat hyperkalemia, treat CHF and pneumonia,  arrange for dialysis.

Purnell      Midgut Volvulus

MGV  can present with vomiting, hyperglycemia, altered mental status.  2 yo with Initial clinical picture that looked like DKA with a blood sugar of 600 and lethargic mental status.  Pt had no ketones in urine.   Abdominal exam when pt was resuscitated demonstrated abdominal tenderness/guarding.  Xrays showed distended bowel loops.  U/S shows free intra-abdominal fluid. Pt ended up with a small bowel resection.  He is awaiting small bowel transplant. 

Nosek   Peripartum Cardiomyopathy (PPCM)

Post-partum patient with SOB/CP/Cough. DDX was mostly between PE and PPCM.  Bedside echo showed poor contractility and BNP was quite high.  Pt started on carvedilol and low dose lasix, lovenox.  PPCM occurs 1 month before or up to 5 months after delivery. 90% occur in the first 2 monts post-partum.   Older patients and multparous patients are at greater risk for this problem.  Treat with b-blockers/lasix/digoxin.  Can’t use ACE-I during pregnancy but ACE-I’s can be useful post-partum.  If while pt is ill EF>30%, it is likely pt will regain good EF.   Girzadas comment: Lung windows can be a hepful adjunct to cardiac echo.  The finding of “headlights in the fog” consistent with extra lung fluid is very reliable.  Chastain comment: U/S findings preceed CXR findings.  Lovell comment: It is true the CXR findings were not impressive.

Conference Notes 3/19

Conference Notes  3-19-2013

Please consider donating to our EM Foundation to benefit resident education/development.  Thanks for your consideration/generosity.
/em-foundation/

Badillo    Ultrasound

Sorry, I missed this lecture.

Lovell   Study Guide  Sedation/Analgesia/Local anesthetic

5 ways to decrease the pain of local infiltration of anesthetic: small guage needle (Harwood uses insulin syringe initially followed by larger meeting), bicarb buffering, warm the anesthetic  (Anneken method: put it on the computer CPU at the beginning of your shift),inject anesthetic  into the wound not through intact skin, slow injection.

Etomidate causes myoclonus up to 30% of the time when used for sedation/induction.

Toxicity of local anesthetics is CNS toxicity (paresthesias followed by seizures) early followed by cardiovascular toxicity (arrhythmias).  Bupivicaine is a great local anesthetic but has the highest risk for toxicity.  Treat seizures with benzos and treat arrhythmias with amidarone.  There is also some case reports on lipid infusions for local anesthetic toxicity. It has been very effective in some case reports.  Risk for local anesthetic toxicity is greater in kids so be careful of dosing and risk also higher if you accidentally inject into an artery/arteriole. 

Sedation levels: Minimal (anxiolysis), moderate (airway reflexes intact), deep (airway reflexes may not remain intact), general anesthesia (airway and cardiovascular issues).

  • Analgesia – Relief of pain without intentionally producing a sedated state. Altered mental status may occur as a secondary effect of medications administered for analgesia.
  • Minimal sedation – The patient responds normally to verbal commands. Cognitive function and coordination may be impaired, but ventilatory and cardiovascular functions are unaffected.
  • Moderate sedation and analgesia – The patient responds purposefully to verbal commands alone or when accompanied by light touch. Protective airway reflexes and adequate ventilation are maintained without intervention. Cardiovascular function remains stable.
  • Deep sedation and analgesia – The patient cannot be easily aroused, but responds purposefully to noxious stimulation. Assistance may be needed to ensure the airway is protected and adequate ventilation maintained. Cardiovascular function is usually stable.
  • General anesthesia – The patient cannot be aroused and often requires assistance to protect the airway and maintain ventilation. Cardiovascular function may be impaired.
  • Dissociative sedation – Dissociative sedation is a trance-like cataleptic state in which the patient experiences profound analgesia and amnesia, but retains airway protective reflexes, spontaneous respirations and cardiovascular stability.  (up to date)

 EMLA is an acronym for eutectic mixture of local anesthetics.

Simple rule: If you want to be cautious, Patient to be sedated should be NPO for 3 hours for any liquid or solid.  If situation is emergent/urgent, you can shorten that time period.  It is controversial whether NPO status does anything to protect patient from aspiration.

Safe dosing for : Lidocaine  4mg/kg plain/7mg/kg with epi      Bupivicaine  3mg/kg plain/5 mg/kg with epi

Can’t use nitrous oxide (NO) in patients with high oxygen requirement,  SBO, pneumothorax, altered mental status.  Nitrous oxide will expand gasses in closed spaces.  Pt’s need normal mental status to cooperate with NO sedation.   Balloon tipped catheters in the body may also expand with the use of NO.

Oral opioids in increasing order of strength: Codeine, morphine, hydrocodone, oxycodone, hydromorphine.    Harwood comment: 100micrograms of fentanyl=1.5mg of dilaudid.   25microgram doses of fentanyl are too small.   Start with 1microgram/kg of fentanyl as a rule of thumb.

Complex regional pain syndrome: Pt  s/p traumatic injury who is getting worse a couple of weeks later.  Painful area may be edematous, allodynia present, diaphoretic, possibly mildly erythematous.  Treat by removing cast, low dose steroids, better pain control, consult orthopedist/trauma.

Benzocaine (hurricane spray, teething gel) and prilocaine can cause methemoglobinemia.

There is research showing that you can treat benign headache with local injections 2 cm bilat laterally to C7, 2 cm deep. Use  1.5 ml of bupivicaine with each injection.  (65% effectiveness for complete resolution of pain)

Abstract:
OBJECTIVE: The primary objective of this retrospective chart review is to describe 1 year's experience of an academic emergency department (ED) in treating a wide spectrum of headache classifications with intramuscular injections of 0.5% bupivacaine bilateral to the spinous process of the lower cervical vertebrae. BACKGROUND: Headache is a common reason that patients present to an ED. While there are a number of effective therapeutic interventions available for the management of headache pain, there clearly remains a need for other treatment options. The intramuscular injection of 1.5 mL of 0.5% bupivacaine bilateral to the sixth or seventh cervical vertebrae has been used to treat headache pain in our facility since July 2002. The clinical setting for the study was an academic ED with an annual volume of over 75,000 patients. METHODS: We performed a retrospective review of over 2805 ED patients with the discharge diagnosis of headache and over 771 patients who were coded as having had an anesthetic injection between June 30, 2003 and July 1, 2004. All adult patients who had undergone paraspinous intramuscular injection with bupivacaine for the treatment of their headache were gleaned from these 2 larger databases and were included in this retrospective chart review. A systematic review of the medical records was accomplished for these patients. RESULTS: Lower cervical paraspinous intramuscular injections with bupivacaine were performed in 417 patients. Complete headache relief occurred in 271 (65.1%) and partial headache relief in 85 patients (20.4%). No significant relief was reported in 57 patients (13.7%) and headache worsening was described in 4 patients (1%). Overall a therapeutic response was reported in 356 of 417 patients (85.4%). Headache relief was typically rapid with many patients reporting complete headache relief in 5 to 10 minutes. Associated signs and symptoms such as nausea, vomiting, photophobia, phonophobia, and allodynia were also commonly relieved. CONCLUSION: Our observations suggest that the intramuscular injection of small amounts of 0.5% bupivacaine bilateral to the sixth or seventh cervical spinous process appears to be an effective therapeutic intervention for the treatment of headache pain in the outpatient setting.

Citation:
Treatment of headaches in the ED with lower cervical intramuscular bupivacaine injections: a 1-year retrospective review of 417 patients.
Mellick LB - Headache - 01-OCT-2006; 46(9): 1441-9
MEDLINE® is the source for the citation and abstract of this record

Diamonds are location of injections.  Circle is the spinous process.

 

Carlson    Oral Board Test Review

Case 1: Shaken Baby syndrome with bilat subdurals.    Crtitical actions:  CT head, get CXR/identify rib fractures, say abuse, take custody of child, consult neurosurgery for subdurals.  A lot of residents missed skin exam showing trauma.

Case 2: Complex orthopedic injury with posterior elbow dislocation and lis-franc fracture dlx.  Elise comment: nice video youtube by Gromis/Fakhori on how to reduce elbow dislocation. http://www.youtube.com/watch?v=mlAOGgocRnk

Case 3: Acute aortic dissection. Critical actions are: appropriate imaging studies, make diagnosis, control BP and tachycardia (labetalol, nitroprusside with esmolol), and consult vascular surgery.  Giving ASA in this case for chest pain is a dangerous action.  Dissections involving the ascending aorta are surgical.

Case 4: Kawasaki DZ.   Critical actions are: Say Kawasaki, obtain CBC, start ASA, discuss need for IVIG, admit patient.    Cardiac involvement with coronary artery aneurysms starts around 1 week of illness.  Criteria for Kawasaki’s: Fever for 5 days, conjunctivitis, mucous membrane changes, cervical lymphadenopathy, rash, involvement of fingers/toes with swelling/rash.

Case 5: Testicular Torsion.  Common pitfalls were not doing GU exam, and not giving prompt pain medication.  Trosion can occur any time during life.  2 peaks: first year and adolescence.  Recent EM Rap said don’t do U/S for torsion, go right to surgery.   For the boards and probably real life, do an U/S prior to sending patient to OR.   Harwood comment: EM Rap is not a peer-reviewed publication.  They are giving a lot of opinion.   As a practicing EM physician in most places in the country, urologists want an U/S prior to taking a patient to the OR.

Case 6: Myxedema coma.  Critical actions: intubate, identify history of under-treated hypothyroidism,  give thyroxine, warm patient, treat with antibiotics for infection.

Case 7: CO poisoning.  Critical actions: 100% O2 and transfer to hyperbaric O2.  CO half life: RA=4 hours, 100%O2=40 minutes,   HBO=20 minutes.

Case 8: Ruptured ectopic.   Critical actions: IV fluids/prbc’s,   consult OB to take patient to OR,  give rhogam for A- blood type.   Methotrexate is not indicated to treat ectopics who have free fluid or hypotension or pain plus multiple other complications.   Girzadas comment: EP’s should have nothing to do with giving or deciding to give methotrexate.  Ectopic management with methotrexate is fraught with great medico-legal risk. 

Chastain   Ocular Ultrasound

Use hi frequency linear probe.   Scan the closed eye through lid.  Have pt do EOM while you scan.

Papilledema: you can measure optic nerve sheath diameter.   Diameter > 5.7 in adults is abnormal. Measure 3 mm from the retina.   Harwood comment: can you use this to identify idiopathic intracranial hypertension?   Michelle, not sure if there is a report in the literature on this.

Retinal detachment:  Retina is always attached at the optic nerve.  If it isn’t then it is more likely to be a vitreous detachment.  If macula is detached (mac-off) this is more likely to require surgery.  Macula is just medial to the optic nerve.

Vitreous detachment:  Looks like clothes in a dryer.   

Vitreous hemorrhage: layered hyperechoic material in globe.

Negro   5 Slide Follow Up

Elderly man on Coumadin who fell at home.  Pt did have a small stroke which likely caused fall.  AAA identified on U/S.  Ct showed no leak.   He had endovascular repair of aneurysm in hospital.

Kmetuk  5 Slide Follow Up

Elderly female with vertigo and vomiting.  Abnormal heel to shin exam and nystagmus.  CT head negative.  Neuro dx’d pt with  vertebral-basilar stroke.   CT is 26% sensitive for diagnosing cerebellar stroke.  HINTS testing:  head impulse test suggests stroke if pt keeps eyes on your nose, nystagmus, test of skew (check eye alignment after covering one eye).  

 

Conference Notes 2-12-2013

Much Thanks to Elise Lovell, MD who was the guest author of this weeks notes!
1. Trale asked about cecal vs. sigmoid volvulus. The "coffee bean" sign on abdominal Xray is useful for diagnosis in both, but in sigmoid volvulus, the bowel loop usually points to the RUQ, whereas in cecal volvulus, the bowel loop points to the LUQ. See images (look at discussion for hyperlink to additional visuals of cecal):
Attached is a review article on cecal volvulus. Sigmoid volvulus is usually seen in the elderly, especially those with chronic constipation or immobility, and those with neuropsych conditions. Cecal volvulus is less common, and can be seen in younger patients. Cecal volvulus is associated with inadequate right colon fixation during embryogenesis, leading to a mobile cecum. Prior abdominal surgery is an associated cecal volvulus risk factor. Both types of volvulus can be seen in pregnancy. Case reports have associated cecal volvulus with marathon running. Cecal volvulus clinical presentation is highly variable, whereas sigmoid volvulus patients usually present with abdominal pain, nausea, distension, constipation. Management for cecal volvulus = surgical. Management for sigmoid volvulus = reduction using sigmoidoscopy.
2. Beware of new/different headache in the pregnant/post-partum patient. They bleed, they stroke, they get hypertensive and seize. Attached is a nice article on post partum headache. There's a differential diagnosis table, as well as a table with a history driven approach to post-partum headache along with appropriate workup recommendations.
3. Finally, with regards to stab wounds to the anterior abdomen, we've reviewed some of the work of Inaba and Demetriades at USC before. Abstract from most recent big study is below. They advocate selective non-operative management of abdominal GSW as well as abdominal stabs. CT abdomen is recommended for all potentially observable GSW abdomen patients, whereas in abdominal stabs, plain Xrays and serial exams are enough if the patient is stable, no evisceration, no retained implement, no bleeding from other orifice, and no peritonitis. Like Dr. Salzman discussed today, need a low threshold for laparoscopy of left sided thoraco-abdominal penetrating trauma even in stable patients due to concern of left diaphragm injuries.
J Trauma. 2011 Feb;70(2):408-13; discussion 413-4. doi: 10.1097/TA.0b013e31820b5eb7.

Selective nonoperative management of anterior abdominal stab wounds: 1992-2008.

Source

Abstract

BACKGROUND:

The use of selective nonoperative management for anterior abdominal stab wounds has evolved into a readily accepted practice. Multiple reports have shown this strategy to be both safe and effective. However, there is a paucity of long-term studies.

METHODS:

A retrospective review was performed of all trauma patients presenting for anterior abdominal stab wounds at a Level I trauma center during a 17-year time period. Primary outcomes were the percentage of patients undergoing an exploratory laparotomy and the negative laparotomy rate.

RESULTS:

A total of 7,033 patients sustained a stab wound with 1,961 involving the anterior abdomen. The percentage of patients undergoing exploratory laparotomy decreased during the study period from 64.8% to 37.6% (overall 45.8%). The negative laparotomy rate decreased from 21.3% to 8.6% (overall 18.7%). The negative laparotomy rate of patients who underwent exploratory laparotomy immediately did not change over time (13.8%), whereas the negative laparotomy rate of those patients who underwent exploratory laparotomy in a delayed fashion decreased from 25.0% to 6.25%. The overall mortality was 1.9%, with 6.2% mortality for patients undergoing an immediate laparotomy, 0.7% for patients undergoing a delayed laparotomy, and 0.0% for patients managed nonoperatively (p<0.04). The mean length of hospital stay was 6.6 days±0.5 days, with a mean of 9.4 days±0.9 days in patients undergoing an immediate laparotomy, 8.1 days±0.5 days in patients undergo a delayed laparotomy, and 3.8 days±0.2 days in patients managed nonoperatively (p<0.001).

CONCLUSIONS:

Selective nonoperative management for stab wounds to the anterior abdomen is associated with a decreased operative rate and decreased negative laparotomy rate over time. Selective nonoperative management is both safe and effective for anterior abdominal stab wounds.

Conference Notes 12-18-2012

Conference Notes 12-18-2012

Last Conference until  1-8-2013.  You can use that time to donate to our EM Foundation to benefit resident education/development.  Thanks and Happy Holidays!
/em-foundation/

Albanis   Ophtho Emergencies

Three key questions: Vision Loss?   Pain in the eye?  Previous Surgery?    Any previous hx of eye surgery puts pt at higher risk for endophthalmitis  or globe rupture.   Both can occur years after eye surgery.  The risk is probably highest following glaucoma surgery.

Visual acuity is the most important thing you evaluate about the eye.   VA is prognostic.  Better VA’s are more likely to do better than patients with worse VA’s for a similar process.

Barounis question: How do you get lids open with a swollen eye?  Albanis and Harwood comments: You need lid retractors or use bent paperclips to retract lids.   Check for 4 basic things: Is the globe grossly intact?   Does the pupil react?   Is there a hyphema?  Is there a 360degree sub conjunctival hemorrhage?  If these 4 are OK then you can move on, there is unlikely to be a severe eye injury.  If any are abnormal you should consult/image.

Case 1: Amaurosis Fugax: painless monocular vision loss, due to an embolism to the retinal artery or a branch thereof.

Stye is an acute staph infection.  Treat with warm compresses/lid scrubs/optional oral antibiotics if you have concerns about a preseptal cellulitis developing.  For styes that are really pointing resist the temptation to incise it with a needle. The ophthalmologists sit on these using warm soaks/lid cleansing/possible abx for 4 weeks prior to surgical treatment.  Concerns about surgery are that it can cause scarring or affect the mebomiun glands.  Chalazion is basically treated the same as stye.

Eyelid lacerations:  ER docs stay away from repairs near the medial canthus, and probably don’t repair lacs that involve lid margins.   Both are high risk for complications.  The general opthomologists don’t even repair lacs around the medial canthus; they refer to oculo-plastics.

Great test to identify myasthenia gravis in a kid with ptosis: Place an ice pack for a couple of minutes on the ptotic eyelid.  When you take the ice pack off and sit the patient up, if they have MG, the ptosis will temporarily resolve.

Subconjunctival hemorrhage in the setting of non-trauma is virtually always benign even if it is 360 degrees.    In the setting of trauma, a 270-360 degree subconjunctival hemorrhage is suspicious for a globe rupture.  If there is any second finding (vision loss/hyphema/altered pupil/etc)in association with this severe subconj hemorrhage, patient will likely go to surgery.   CT can help in this situation.

Barounis/Chastain    STEMI Conference

CASE 1: Posterior MI: Diagnosis is difficult because no specific leads represent this area.   You need posterior leads to verify the infarct (post leads are specific but not sensitive).   ST segment depression V1-3.  Prominent R wave V1-3.  Posterior MI frequently co-exists with acute inferior or lateral MI.  The vessel commonly involved in an isolated posterior MI is the circumflex.   If you are uncertain whether patient has an AMI, stat 2D Echo can help identify wall motion abnormality.   Harwood comment:  this patient had an isolated circ lesion causing isolated posterior mi.   Pt partially reperfused his vessel with his own TPA accounting for his improved 2nd EKG and temporized his clinical status until he was taken to the cath lab.

Case 2:  75 yo male with chest pain and new LBBB.  Harwood comment: pt has new LBBB and new first degree AV block putting him at risk for complete AV block.    Comment from care coordinator: If you document new LBBB on the first EKG in the chart, that case goes into the STEMI care bundle.  If you don’t feel the patient has a STEMI you need to document why the patient is not going to cath lab.    The STEMI Care bundles don’t factor in Sgarbossa criteria.    Proposed management for suspected ACS with presumed new LBBB: if unstable or CHF go to cath lab.  If stable with Sgarbossa criteria present, go to cath lab.  If Stable with no sgarbossa criteria present get stat echo.  Simplified Sgarbossa criteria are concordant st elevation >1mm or concordant st depression V1-3 >1mm,   1 positive Sgarbossa criterion is 95% specific but not sensitive.   The discordant ST elevation of 5+mm anteriorly has been taken out because of a relatively low likelihood ratio.

Case 3: Pt had a right coronary artery lesion.  He had waxing and waning perfusion resulting in intermittent accelerated idioventricular  reperfusion rhythm.  

Barouinis/Chastian/Harwood  Flash Mob Journal Club     

Article discussing oxygen toxicity due to post arrest resuscitation.

Background: Therapuetic hypothermia post-arrest is thought to prevent reperfusion oxidative injury.  Patients have some brain edema post arrest.  High oxygen exposure to the brain  during resuscitation is thought to increase oxidative injury by the production of excess free radicals.

Methods: 173 patients treated with therapeutic hypothermia.    Retrospectively the investigators identified the highest PAO2 recorded for each patient in the first 24 hours post arrest.   They correlated this PaO2 measurement with survival and  functional neuro outcome.    All patients were in the target therapeutic temperature range.

Outcome: Survivors had lower PaO2 compared with nonsurvivors.   Higher PaO2 associated with worse neuro outcome.   Cut off seems to be a PaO2 of220.   Above that level patients did progressively worse with higher PaO2 level.

Potential confounders:  High PaO2 could be just a surrogate marker for a sicker patient or poorer quality of the resuscitation.   The resuscitation team may have had to resuscitate the patient more aggressively and were hesitant to dial down the O2 or the resuscitation team may be subpar and were not watching the oxygenation level closely to optimize patient care.

What to do: The emergency physician may want to closely watch oxygenation and avoid hyper-oxygenation in the post arrest patient.  Consider  starting your vent settings for the post arrest patient at 50% FIO2.   Harwood comment: think of oxygen as a medication and there is a correct dose of oxygen for the post-arrest patient.  Giving boat loads of oxygen after resuscitation and damaging a patient’s brain is analogous to giving too much tobramycin and bagging a patient’s kidneys.    

Animated discussion started by Sola’s question and response lead by Harwood and Brian Febbo about supplemental oxygen in other types of patients.    No benefit of supplemental oxygen above 96% room air sat for  sickle cell patients.   Cochrane review shows harm to providing supplemental oxygen to acute MI patients with room air sats over 96%.

Badillo   Chest U/S

Ultrasonographic A lines are naturally occurring artifact lines or air lines.  They are horizontal on the screen and are stacked on each other.    B lines are bad lines and are signs of interstitial edema.  They are vertical  and extend down to bottom of screen.  3 or more B lines are consider pathologic.  The more you see the more specific the finding.

Parasternal long view:  Kinesis assessment by looking at whether the mitral valve anterior leaflet hits the septum and do the walls of the LV almost come in contact in systole.

Case1:  CHF patient with poor kinesis on echo and B line comet tails that signify pulmonary edema.

Case 2: US is better than supine CXR for identifying pneumothorax.  Use the linear probe in midclavicular line in area of 2nd intercostals space.   Look for sliding pleura between rib shadows.  There may be small B lines(also called minor B lines, comet tails, Z lines)  that don’t go all the way down to the base of the screen and are normal artifact as opposed to the abnormal B-lines that signify CHF.    There is an M-mode view that can also identify pneumothorax.  Patients with a pneumothorax have a “barcode” type appearance.

Big RV with bowing of septal wall toward LV raises concerns for PE.

Practical Scanning with the Residents.

Applicant Review

Conference Notes 12-11-2012

Please consider a gift to our EM Foundation to benefit resident education/development.  Thanks!
/em-foundation/

 

Conference Notes 12-11-2012

Walchuk/Watts   Oral Boards

Case 1 Esophogeal variceal bleeding:  Fluid/blood resuscitation.  Manage airway as patient becomes confused.  Protonix drip.  Octreotide 50microgram bolus followed by 25-50mics/hr.  Consult GI.  Could consider IR for TIPs procedure.   Correct any coagulation defect with FFP or Factor 7A. Give rocephin or cipro to avoid infectious complications and limit re-bleeding.   Endoscopy with sclerotherapy or banding teamed up with Octretide has better outcome than either endoscopy or octreotide alone.

Case 2 Organophosphate Poisoning: IV hydration, decontaminate, give Atropine and 2-PAM.   Usually need large doses of atropine. SLUDGE BBB=salivation/lacrimation/urination/diarrhea/gastric emptying/bronchospasm/bronchorhea.  Usually due to insecticide poisoning.  Most common pitfall is inadequate atropinization.

Case 3 Roseola infantum:  Also known as sixth’s disease/exanthema subitum/three day fever.  Patient is not contagious at the time of rash appearance.   Non-pruritic Rash appears after 3-5 days of fever.  Pt is well appearing.      Erythema infectiousum (fifth’s disease) will have slapped cheeks and lacey extremity rash.  It is usually in school age children.

Elise comment: Utilize massive transfusion protocol for GI bleeders in shock.  Could also consider FEIBA for coagulopathy.  Get 2 IV’s or a cordis in these patients to give large volume crystalloid/blood product replacement.   Any tox case on oral boards call poison control.   Figure out the OD by talking to family/friends/pre-hospital personnel.

Harwood comment:  Start to mobilize atropine 10-50 amps for treatment of organophosphate poisoning as soon as you suspect the diagnosis.   You will burn through all the atropine in the ED.  

Ketanneh  Trauma

The NEXUS decision instrument stipulates that radiography is not necessary if patients satisfy ALL five of the following low-risk criteria:

  • §  Absence of posterior midline cervical tenderness
  • §  Normal level of alertness
  • §  No evidence of intoxication
  • §  No abnormal neurologic findings
  • §  No painful distracting injuries

Insignificant injuries were defined as those that would not lead to any consequences if left undiagnosed. The NEXUS investigators evaluated 34,069 blunt trauma patients who underwent radiography of the cervical spine comprised of either a three-view cervical spine x-ray or a cervical spine computed tomography (CT) scan. Of these patients, 818 (2.4 percent) had sustained a cervical spinal column injury. Sensitivity, specificity, and negative predictive value (NPV) of the NLC were found to be 99.6 percent (95% CI 98.6-100), 12.9 percent (95% CI 12.8-13.0), and 99.9 percent (95% CI 99.8-100), respectively. (These notes cut/pasted from Up to Date)

Tenderness should have objective signs like physical evidence of discomfort.

Altered level of alertness: GCS<15, disorientation, Inability to remember 3 objects at 5 minutes.

Painful distracting injury: Undefined and varies by physician. 

 Canadian C-Spine rule

The CCR involves the following steps:

  • §  Condition One: Perform radiography in patients with any of the following:
    • ·         Age 65 years or older
    • ·         Dangerous mechanism of injury: fall from 1 m (3 ft) or five stairs; axial load to the head, such as diving accident; motor vehicle crash at high speed (>100 km/hour [>62 mph]); motorized recreational vehicle accident; ejection from a vehicle; bicycle collision with an immovable object, such as tree or parked car
    • ·         Paresthesias in the extremities
  • § 
    • ·         Simple rear end motor vehicle accident; excludes: pushed into oncoming traffic; hit by bus or large truck; rollover; hit by high speed (>100 km/hour [>62 mph]) vehicle
    • ·         Sitting position in emergency department
    • ·         Ambulatory at any time
    • ·         Delayed onset of neck pain
    • ·         Absence of midline cervical spine tenderness

Patients who do not exhibit any of the low-risk factors listed here are NOT suitable for range of motion testing and must be assessed with radiographs.

If a patient does exhibit any of the low-risk factors, perform range of motion testing, as described in Condition Three below.

  • §  not

In the derivation study, the CCR demonstrated a sensitivity of 100 percent and a specificity of 42.5 percent for identifying clinically important cervical spine injuries [22]. In 2003, the CCR was prospectively studied in the emergency departments of nine Canadian tertiary care hospitals. Of 8283 patients, 162 were found to have clinically significant injuries, and the sensitivity, specificity, and negative predictive values of the CCR were respectively 99.4 percent (95% CI 96-100), 45.1 percent (95% CI 44-46), and 100 percent [23]. The investigators reported that the CCR would have missed one patient with a clinically important cervical spine injury, while the NLC would have missed 16. The CCR has also been validated in larger hospital-based studies and in an out-of-hospital study of paramedics [24,25].  (These Notes cut and pasted from Up to Date)

Negative CT Cspine with persistent pain:  3 management options: continue collar with f/u with neurosurg.  Get MRI if neg then remove collar.  Get the dreaded flex-ext cspine films.   The utility of Flex/ext films is controversial.

Negative CT Cspine and Obtunded:  Don’t do flex/ext films.  Do MRI.   One study found 9% rate of abnormal MRI in obtunded patients with negative CT Cspine.   If you need to leave patient in collar for a prolonged time, use an Aspen Collar.

EAST guidelines were discussed and are basically a combination of NEXUS/Canadian rules with the above recommendations for what to do with the patient with negative CT cspine and continued concern for injury.  (If you want more info you can find the EAST guidelines at east.org)

 
 
 
 
 
 
 

Girzadas/C. Kulstad/Harwood comment:  Agreement  of commenters that there is still some utility to do a Cspine plain film series in a low risk/young patient.  You avoid excess radiation exposure and still can see C1 to C7. In a patient with higher risk or any increased difficulty in getting good quality plain films go right to CT Cspine.   Dr. Smith (Trauma) felt strongly you need a CT to rule out Cspine fracture.

Barounis comment: If you are working in a community ED and are transferring a severely injured patient that is obtunded, get a cspine film/cxr/pelvis xray.

Harwood/Smith comment: Don’t do flex/ext films.  Go right to MRI in the patient with significant neck pain or obtunded and negative CT Cspine.

Gottesman  M and M

25yo male with GSW to right face.  CT head showed facial fractures.  No intracranial injury. No cspine injury on CT Cspine

Lefort Fractures:  Type 1 fracture line goes through alveolar ridge and allows the front teeth to move with traction.    Type 2 fractures allow the nose to move.  Type 3 fractures allow the zygomas to move.

Patient in case became agitated in CT and developed hypoxia.  Trauma team moves to attempting intubation.   Glidescope intubation was successful.

Avoid nasal intubation in patients with facial fractures.

Pt developed increased bleeding from facial wound.  Wound/nose/oral cavity were all packed.

Causes of agitation: Head injurying/toxins/infection/hypoxia/hypoglycemia/intracranial injury/stroke/seizures/emotional stress/sensory deprivation.  Psychiatric cause is basically a diagnosis of exclusion.

Pt was found to have a high Internal Carotid Injury on CTA.  Went to OR but pt did not have surgery because he had an injury that the surgeons could not get to.   His injury was zone 3.

Harwood comment:  sometimes these high zone 3 injuries require surgical removal of mandible during the time of surgery.  Patients that have this procedure frequently have difficulty with eating and talking the rest of their lives.

Elise and Harwood comment: If you see this patient in a community ED, Tube/get facial film/cross table lateral cspine film/cxr prior to transfer.  

Patient in Case blows a pupil.  Pt stroked from left internal carotid injury and developed herniation

Take home point: Beware the agitated patient.  Think of hypoxia and other serious causes prior to attributing to psych reasons.

Harwood comment: As soon as you decided this patient has to go to CT you should have intubated him.

Garrett-Hauser     Ethics   Mandatory Reporting

Case 1:  Female patient with AIDs was having sexual relations with a man who did not know her HIV status.  They use condoms during intercourse.   Do we have obligation to notify her partner?

There is a law on the Illinois’ books titled Criminal Transmission of HIV.  Person has to know they have HIV and have sex without a condom with a person who does not know they have HIV.   Prosecution requires a court order for release of medical records.   So we don’t need to notify the police.     Harwood comment: If you know that HIV person knowingly    has sex with partner with no condom, do you notify the non-HIV partner?   He would first try to convince the HIV patient to notify the non-HIV patient.  Dr. Garrett-Hauser made a strong point that it likely would be a HIPPA violation to call the police.  There was a animated discussion among residents and attendings about whether and  how to notify the partner at risk.   Elise and Harwood would notify the at-risk partner.   C. Kulstand and Girzadas would give the at-risk partner a more obtuse warning such as: As a physician, I would advise you not to have sexual relations with that person until you have a detailed conversation with that person.

Case2: Intoxicated parents bring child to ER. Don’t let parents drive the child home.  Notify DCFS.

Case 3: 14yo female found to be pregnant at PMD’s office.  Pt’s mom brings child to ER that same day for U/S and was threatening lawsuit because the 14yo was in ER one week ago and diagnosed with pregnancy  and mom was not told.   ER records from previous ER visit documents that 14 yo patient refused to allow staff to notify mom of her pregnancy.     Shayla discussed that the decision to notify parents of medical condition of a minor is a balance of patient’s cognitive maturity and risk of medical condition.   The state law for emancipated minors specifically gives emancipated minors the right to consent for treatment  but not specifically the right to privacy.   Discussion of risk came down to: the parent is much more likely to sue you for not notifying them than the young patient is to sue you for ratting them out.

Case 3: Illinois does not have a mandatory reporting law for domestic violence.  The only state with such a law is California.  In California, ED physicians are mandated immediate reporters.  These laws are controversial because there has been increased morbidity or mortality for patients in which MD’s reported before the patients were personally ready to have abuse reported.

RLT Applicant Review

Procedural Tip: when performing a digital block, wait 10-15 minutes to optimize anesthesia.  Tell the patient you are going to see another patient and will be back in a few minutes. Don’t leave them too long though.  It’s embarrassing to have to anesthetize them twice.   I like to use bupivicaine without epi because it lasts longer than lido. 

Conference Notes 11-27-2012

Conference Notes 11-27-2012

Putman/Coghlan    Oral Boards

Case 1: Pt with TIA 8 days after AMI.  Pt had a persistent ventricular aneurysm with likely mural thrombus.   Dr. Coghlan said heparin would be indicated for mural thrombus.   Mural thrombus develops in the first week after AMI and embolizes in the second week. 

Case2: Child with sickle cell disease and splenic sequestration crisis.  Have to assess for splenic rupture. Give IV fluids and transfuse 10ml/kg PRBC’s.   Mortality in splenic sequestration crisis is 12%.

Case3: Child swallowed a button battery.  Battery was identified in stomach. Management was to observe as outpt with serial xrays.  

Elise comments: Fluids and O2 in sickle cell is no longer universally indicated.  If spleen is enlarged you know you will need a blood transfusion.   NL systolic BP in Kids is 2 X age plus 70.   If button battery is ingested with magnets it increases risk of perforation. 

Harwood comment: If button battery is not making progress on serial xrays, the kid will need a scope.  Transfusing blood in kids with shock you can give 20ml/kg of PRBC’s.

Coghlan comment: Sharp objects such as needles need to be removed.

Group discussion about heparin for TIA with possible mural thrombus.  Everyone agreed that this was a difficult/ controversial situation.  You would like to have an echo and have neuro and cardiology consults to help make this decision.

RLT/Residents  Applicant Review

Discussion of the progress of recruiting season.

Schroeder  Visual Diagnosis in Pediatric Patients

Paronychia with lymphangitis.  Lymphangits is associated with Group A strep infection.  In young kids, admit for iv antibiotics.  Older, well appearing kids can go home on po antibiotics.

Mucocoele on inner lip.  Blockage of minor salivary gland due to biting lip.  Can refer for excision.

Mastoiditis: Trend to do less ct’s and manage clinically with antibiotics.

Salmon colored debris in diaper is urate crystals.  Young kids excrete more urate crystals. It is benign. Not associated with infection or other problems.  

Thick white exudate on a 2yo tongue=thrush.  Could be due to inhaled steroids.  Could be due to having child sleep with bottle in the mouth.

Erythema migrans from lyme disease.  Circular erythematous rash. Pt had tick in hair.  Lyme disease is increasingly found in Illinois counties.

Herpetic whitlow can be complicated by Group a strep causing lymphangitis.  Herpetic whitlow also uncommonly can have signs of lymphangitis due to the virus itself.  Harwood comment: I would treat with both anti-strep antibiotic and antiviral medication.

Eruption hematoma is basically a blood blister associated with an erupting tooth.  Nothing to do about it.

Kids can have a sucking blister on their lips from sucking on their lips.  It can look like cyanosis in darker skinned children.

Bed bug bites result in a rash that is nonspecific.  With itchy macular-papular rashes think scabies and bed bugs.   Families with bed bug infestations need to consult a professional company. 

Aphthous ulcers are caused by T-cell immunity.   Treat with mouthwash rinses or oral steroid gel.

Cobblestoning of the posterior pharynx can be due to post nasal drip.  Treat with nasal steroid or Sudafed.

Bullous impetigo can be treated with mupirocin topically.  Can treat more extensive infections orally with bactrim and keflex.  There have been MRSA strains of impetigo isolated.  Treat all impetigo as potential MRSA infection.  Mupirocin covers MRSA.  If using oral abx: bactrim/keflex or clinda to cover both MRSA and strep. 

Eczema herpeticum is a disseminated HSV infection.  Most often in kids with bad eczema. Treat with IV acyclovir.    Also give iv antibiotics  for possible severe impetigo until cultures clarify diagnosis.  You can use the new cultures we have in the ED for both bacterial and viral cultures.

Unilateral parotid swelling is more likely to be bacterial so treat with abx.  Bilateral swelling is more likely viral.   Harwood comment: with parotid swelling from infection the ipsilateral ear will pop out somewhat like mastoiditis.  This can be a clue to help you localize the swelling to the parotid.

Bill showed pictures of hand foot mouth.  The vesicles with HFM are thick walled, flat and grey compared to chickenpox vesicles.   Also Chicken pox won’t affect the palms and soles.  HFM can have lesions on more proximal extremities and face in addition to hands and feet.  There are seasonal variations to the severity of HFM.  In Cambodia there was a fatal form of HFM.  Bill felt that fatal cases have been localized to Asia and we should not alter our practice of discharging most patients in the US at this time.

Kerwin  Atrial Fibrillation

Who gets afib: age, htn, coronary heart disease, valvular disease, chf, HCM, congenital heart disease, obesity, dm, thyroid disease, CKD, cardiac surgery,  family hx, ETOH (holiday heart syndrome), PE, sleep apnea, numerous medications cause.

A-fib increases risk of all-cause mortality, CHF, and stroke.  

Paroxysmal AFIB lasts less than 7 days.  Persistent AFIB  last longer than 7 days. Permanent AFIB is afib in which attempts at cardioversion failed.   Lone afib is in patients <60 with no cardiopulmonary disease including HTN.

Get usual labs and TSH to work up. Get troponin as part of  work up only if ischemia is suspected.

NEJM study 2002 showed no survival advantage of rhythm control vs. rate control. Multiple other studies confirm this finding.  But in 2012 study in Circulation showed decreased rate of stroke in rhythm control patients.

ED management: Divide patients into 3 categories.  Pts who are unstable.  Pts with afib less than 48 hours.  Pts with afib more than 48 hours.  

Afib >48 hours: Rate control and anticoagulation are management goals.  No clear data on superiority of beta-blocker or calcium channel blockers for rate control.   No benefit from strict rate control (<80) vs. lenient rate control (<110).   For patients with heart failure consider digoxin or amiodarone instead of beta blocker/ca channel blockers.

CHADS2: CHF, HTN, Age>75, DM, Prior stroke or TIA. This tool assesses stroke risk with afib.  With a CHADS2 score of  0 or 1 you can opt to treat with ASA and not warfarin. 

Hemodynamically unstable patient: Cardiovert.  Anterior-posterior pad positioning is optimal.  Higher energy doses are more effective.  Cardioversion does not cause myocardial injury so higher dose is not really a concern.  If you have to shock more than once, wait at least a minute between shocks.  Consider giving push dose pressors to support BP prior to cardioversion.  1ml of phenylepherine in a 100ml bag of saline.  Mix and draw up 10ml of this solution. Can give 1-2 ml every 5 minutes.  

Afib<48 hours: Can attempt rhythm control. It avoids admission and meds.    Ottawa aggressive protocol includes procainamide 1gm over 60 minutes.  If that doesn’t work sedate and cardiovert with 150j. Converted patients discharged on no meds.  Multiple studies seem to show No increased risk of death or stroke from early cardioversion protocol.  About 8% of patients had 7 day recurrence.   Another consideration is that the rate of spontaneous conversion in new onset afib patients is up to 70%!  So centers will send patients home and ask them to return in 24 hours for a recheck.

Afib with WPW use cardioversion if unstable. Use procainamide if stable.

Afib in pregnancy: CCB’s , Beta-blockers, and digoxin are all ok. Cardioversion is ok if unstable. IV heparin is ok.

Harwood comment: Treating unstable afib patients use highest dose possible of cardioversion.  If not effective, wait a minute or two and try again.   If second shock is ineffective, give procainamide over 30 minutes and retry shocking.  Get cardiologyto help you if the first two shocks don’t work.  If you need to support BP give push dose pressors or pressors on drip.

Barounis comment: How do you manage the septic patient with afib?  Harwood responded to give digoxin.  Shock if necessary but most of these patients are in afib chronically.  Other attending suggested IV magnesium or amiodarone as alternative approaches.

Conference Notes 11-20-2012

Conference Notes   11-20-2012

Chandra    Thoracic Aortic Dissection

Tupak Shakur is still alive! (Urban myth)

Stanford Type A involves ascending aorta.  Type B Stanford is only descending aorta.

Two different anatomic systems, the DeBakey and Daily (Stanford) systems, have been used to classify aortic dissection [2,23-25]. The Stanford system is more widely used. It classifies dissections that involve the ascending aorta as type A, regardless of the site of the primary intimal tear, and all other dissections as type B. In comparison, the DeBakey system is based upon the site of origin with type 1 originating in the ascending aorta and propagating to at least the aortic arch, type 2 originating in and confined to the ascending aorta, and type 3 originating in the descending aorta and extending distally or proximally.

Ascending aortic dissections are almost twice as common as descending dissections. The right lateral wall of the ascending aorta is the most common site of aortic dissection [1]. In patients with an ascending aortic

 

6.4% of patients have been found to have painless aortic dissection.  Most of the time these are ascending dissections that present with stroke or syncope.

In patients with shock in the setting of suspected aortic dissection, think pericardial tamponade.  The dissection can bleed into the pericardial sac.

Diagnosis:  Ekg is mostly non-specific but can have ami in 5% of patients with dissection.  CXR can show wide mediastinum.  CT angio has sens/spec in the mid to high 90%’s.   MRI has the highest sens/spec 98%!  Aortography has sensitivity of 88% and specificity 94% so it is no longer used.  TEE has a sens/spec of about 98%.   This is probably the way to go for the unstable patient or the patient with high risk for aortic dissection who can’t take contrast.  Harwood comment: If you aren’t sure if the patient has a PE or dissection, get the CT PE protocol because it is harder to identify PE than dissection.  Harwood and Elise both agreed that if you suspect one diagnosis over the other, get the test specific for that diagnosis.   The timing of the dye load is different for both tests and if you get the CT PE study it decreases your sensitivity for dissection.  The CT test for dissection lowers your sensitivity for pe.  

Management:  BP control start with esmolol and follow with nitroprusside.  You want to decrease shear stress on the aortic wall.   Labetalol is another good single angent choice.   Maletich comment: How bout esmolol and nicardipine?  Paarul/Harwood/Elise al felt this was reasonable as well.   Type A dissections are managed surgically.  Type B dissections are managed medically unless they have signs of ischemia, progressive dissection, intractable pain,impending rupture, or uncontrolled hypertension.   Harwood comment: You have to advocate for the patient and get them to the OR with descending dissections when they have one of the above complications.

Poor prognosis: advanced age, hypotension, renal failure, pulse deficit, need for massive transfusion, many others.

Girzadas comment: Some respected authorities say It is the standard of care to miss aortic dissection.  Harwood comment: Aortic dissection is the hardest diagnosis in the world.   Harwood’s own decision rule for dissections is classic aortic pain get ct angio.  If pain is not classic but you still have suspicion: check bp’s in both arms if abnormal get a CT angio,  if chest pain and abnormal cxr get CT angio,  if chest pain and abnormal d-dimer get a ct angio.   Coghlan comment: remember this diagnosis in pregnant patients and patients who had a recent coronary angio.   Gupta comment: this diagnosis is rare and difficult. It should be considered ok to miss this diagnosis.   Work up for dissection delays diagnosis of other more common diagnoses and these patients generally do poorly.   Chastain comment: gotta look at your own CXR’s because you have a different perspective on the mediastinum than the radiologist.

Putman     Neck Trauma

 

 

Harwood comment: Intubate early to avoid losing the airway.   The risk of losing airway due to expanding hematoma is much greater than an early intubation.   Doing a cric in a patient with neck trauma is very difficult.    Lovell comment: You have one shot to get this airway.  Go right to most reliable device which may be glidescope.

Prior to RSI, check for neuro deficits especially cranial nerves.   You only need to immobilize cspine in patients with blunt trauma and/or neuro deficit.   Isolated penetrating neck injury does not require cspine immobilization.

Penetrating Trauma Hard signs: hypotension, arterial bleed, decreased carotid pulse, expanding hematoma, thrill or bruit, lateralizing signs, air bubbles in wound, hemoptysis/hemetemesis, tracheal deviation.  These all go to the OR.

Penetrating Trauma Soft signs: hypotension in field, history of arterial bleed, bradycardia, large hematoma, apical cap on CXR,  stridor, hoarse voice, subQ air,  CN7 deficit.  These patients all require further evaluation.

Zone 3 injuries need CT head.  

Helical CTA is the work horse study for neck injuries.  It shows vascular structures as well as cspine and aerodigestive tract.    Harwood comment: Selective angiography is very rarely done.  CTA is the way to go all the time, unless there is metallic FB’s or hardware obscuring the CTA image.

If CTA is neg then patients should get CXR, EGD and contrast esophogram.  Also they get laryngoscopy/broncoscopy.    Zone 3 injury gets catheter angiography.

Blunt Trauma

Wallenberg syndrome:  Can get from vertebral artery injury

Common Symptoms

  

 

Ipsilateral facial pain and numbness

  

 

Vertigo, nausea, and vomiting

  

 

Ipsilateral clumsiness

  

 

Diplopia, oscillopsia

  

 

Numbness ipsilateral or contralateral to lesion

  

 

Dysphagia, hoarseness

Signs

  

 

Contralateral:

  

 

Impaired pain sensation over half the body, sometimes including the face

  

 

Ipsilateral:

  

 

Impaired sensation over half the face

  

 

Ataxia of limbs, falling to side of lesion

  

 

Horner syndrome

  

 

Dysphagia, hoarseness, paralysis of vocal cords

  

 

Diminished gag reflex

  

 

Loss of taste

  

 

Other:

  

 

Nystagmus:

  

 

Primary-position rotatory

  

 

Gaze-evoked horizontal

  

 

Downbeating on lateral gaze

  

 

Ocular skew deviation

  

 

Hiccup

 

E Kulstad   CV Study Guide

Asa has 4% absolute mortality benefit from 13% to 9% in AMI (25% relative risk reduction).   ACMC’s overall AMI mortality is 15.2% which is lower than the national norm.

Bening Early Repol: widespread ST elevation (precordial greater than limb leads), J point elevation, concave up st segment, notching of J point,  prominent concordant t waves.   Girzadas comment: BER should not have reciprocal st depression

Electrical alternans is a sign of pericardial effusion.  Low sensitivity/high specificity.

 

Electrical alternans   Every other QRS complex has a reduced amplitude alternating with a higher amplitude

 Stress testing:  Sensitivity/specificity is relatively low for coronary stenosis.  Post-test probability = pretest probability X LR.  The LR for stress testing is 0.25.  So if you have a patient with 10% risk of coronary stenosis and he has a negative stress test, you bring his risk down to 2.5%. It is debatable about whether this is low enough for feel comfortable with.

Anterior wall AMI with new LBBB and first degree AV block: Pt should get stand by prophylactic temporary pacemaker.  (Zoll Pacer Pads)   

Plavix has not been shown to significantly decrease mortality  in ACS.  We use it because the composite end point of death/stroke/non-fatal mi is lower than placebo.  This outcome has been accepted by our cardiology colleagues as beneficial for patients.

Heparin has also not been shown to lower mortality in ACS compared to placebo in a Cochrane review. Heparin has been shown to lower the risk of subsequent AMI in ACS.  Cochrane review showed no difference between heparin and LMWH.  As of today amazingly, the Cochrane review has not been concluded on heparin for STEMI.

History of CHF is a marker for risk of sudden death following syncope.

Beta-blockers and ACE-I’s and spironolactone have been shown to reduce mortality in CHF.  Lasix has not been shown to decrease mortality.

High dose IV NTG has been shown to improve outcomes in acute CHF exacerbations.

CHF mortality is 50% in 4 years. 

Contraindications to TPA in AMI:  Prior ICH/Intracranial malignant tumor/structural cerebral vascular lesion/ischemic stroke in last 3 months/significant closed head trauma in last 3 months/active bleeding or bleeding diathesis.

Smoking cessation has the best mortality benefit of any intervention in CHF.

Sgarbossa Criteria:

The three ECG criteria with an independent value in the diagnosis of acute infarction and the score for each were:

  • § 
  • § 
  • § 

A minimal score of 3 was required for a specificity of 90 percent. The first two criteria are similar to those described above since the ST segment is concordant rather than discordant with the QRS complex. However, the third finding requires further validation, since a high take-off of the ST segment in leads V1 to V3 has been described with uncomplicated LBBB, particularly if there is underlying left ventricular hypertrophy. In a substudy from the ASSENT 2 and 3 trials, the third criteria added little diagnostic or prognostic value [15].  Harwood comment: It is not helpful in the vast majority of patients with LBBB. But when you have it is pretty specific.

Modi  LVADS

Who can receive LVAD:  Class 4 heart failure on max medical therapy, people who can’t come off bypass, cardiogenic shock, bridge to transplant.

Only 24 heart transplants in Illinois this year.  112 patients on destination therapy LVADS.  Longest living patient on LVAD at ACMC is 8 years.

LVAD patients do not have a pulse. The pump is not a pulsatile pump, it is continuous flow.  Probably more directly a cause is that the aortic valve doesn’t open with this pump.  The pump  connects the LV to the aorta.  There may be more intracranial hemorrhages due to continuous laminar flow.   Newest pump has a magnetically levitated internal rotor device so there is no wear on the rotor internally.

Initial eval: check for fever.  Patients are at risk for infection with drive tube in abdomen.  Most patients will not have a pulse. No affect on respirations.  Blood pressure is usually around systolic of 90-120. Ekg is unaffected by LVAD.   Pt’s may have VT while on the LVAD but will still have the pump working.  Patients will be alert and talking with VT.  You don’t need to shock patient if they are alert and talking and not in shock.  Treat the arrhythmia with meds until pt has altered mental status/dyspnea/low bp.  When evaluating a patient, check the computer reading, it will likely tell you of any problems with the device.   If pt is unresponsive and arrives in the ED with a disconnected drive line, hook it back up.  You run the risk of throwing a clot from the heart but the alternative is death for the patient.   Chest compressions are ok for unresponsive patients with LVADs.  Check the LVAD and make sure it has power.   Otherwise perform resuscitation as you normally would. 

Lovell Population Health

Population health aims to improve the health of a population.  Reduces healthcare inequities.  Aims to improve the overall health environment of a population.

Accountable Care Organizations and similar models focus on quality, preventative medicine approaches and shared savings instead of trying to increase the volume of healthcare delivered.   Avoid unnecessary care and avoiding complications.  We are accountable for all the patients in our catchment area. 

This will be a new paradigm that we will be part of moving forward in our careers.

Erickson   Eye Emergencies

Find out if the patient uses glasses or contacts.

Get a visual acuity of the affected eye!!  Vital sign of the eye. 

Chemical injury:  Irrigate the heck out of the eye.  Use topical anesthetic prior to irrigation. Check ph pre and post irrigation.

Acid causes coagulation necrosis.  Coagulation necrosis actually prevents deeper penetration of acid into tissue.

Alkali causes liquefaction necrosis which is more serious and can penetrate more deeply into tissue.

Acute angle closure glaucoma: Precipitated by papillary dilation.  Eye pain and emesis/headache.  Hazy cornea and fixed mid-dilated pupil.   Tx with beta blocker, alpha agonist, pilocarpine and mannitol and acetazolamide.  Mnemonic: E=m2c:  save the eye=miotics (pilocarpine/alpha agonist/beta blocker) and mannitol,  carbonic anhydrase inhibitor.

Ruptured globe: eye pain, decreased visual acuity, teardrop shaped pupil, positive sidel’s test. CT orbits can be helpful.  CT may show flat tire sign.

Corneal abrasion: topical antibiotics, consider a cycloplegic agent.  Don’t give them topical anesthetic for home.  Update tetanus shot.

Linsey went through a number of pattern recognition slides of eye emergencies.